SlideShare une entreprise Scribd logo
1  sur  35
Infectious diseases of the skin
1) The causative organism for Brucellosis is which of the following:

A. Acid fast bacilli

B. Gram positive cocci

C. Gram negative rodCorrect Choice

D. Gram positive rod

E. Gram negative cocci
Brucella species which cause Brucellosis are gram negative rods. Brucellosis, aka undulant fever, is
characterized by an acute febrile illness with headache and joint pain. CNS and cardiac
manifestations can also occur. It is acquired by contact with infected animals or contaminated dairy
products. Treatment is with doxycycline and rifampin


2) What is the most likely diagnosis of this lesion occurring on the hand of a butcher?

A. Heck's disease

B. Butcher's wart

C. Contagious ecthymaCorrect Choice

D. Molloscum contagiosum

E. Ecthyma gangrenosum
Contagious ecthyma, or orf, occurs most frequently among shepherds and butchers as it is a viral
disease that is more common among sheeps and goats. It is caused by a farmyard pox, which also
cause Milker's nodules. The lesions tend to heal spontaneously after several weeks


3) For which of the following smallpox vaccination complications is the administration of vaccine
immune globulin indicated?


A. Vaccinia keratitis

B. Mild generalized vaccinia

C. Post-vaccinal encephalitis

D. Eczema vaccinatumCorrect Choice

E. Erythema multiforme
Vaccinia immune globulin can be administered to treat some of the complications associated with
vaccinia vaccination. Vaccinia immune globulin is indicated for inadvertent inoculation with
extensive involvement or ocular implantations, eczema vaccinatum, generalized vaccinia (severe or
recurrent), and progressive vaccinia. It is not recommended for inadvertent inoculation with mild
reaction, generalized vaccinia (mild or limited), post-vaccination encephalitis, and isolated vaccinia
keratitis


4) What characteristic color is seen on diascopy of this lesion?

A. Coral red

B. Orange

C. Blue




                                                   1
D. Red

E. Apple jellyCorrect Choice
Leishmaniasis recidivans is a type of Old World leishmaniasis. Clinically, it appears as a red papule
covered with white scale. On diascopy, it has a characteristic apple jelly color


5) Measles is caused by a:

A. Togavirus

B. Rhabdovirus

C. Picornavirus

D. ParamyxovirusCorrect Choice

E. Adenovirus
The measles virus is an RNA virus belonging to the Paramyxovirus family


6) Contagious pustular dermatitis (Orf) is caused by a:

A. Poxvirus Correct Choice

B. Gram-positive spore-forming rod

C. Papovavirus

D. Herpesvirus

E. Paramyxovirus
Orf is caused by orf virus (OV), a poxvirus of the genus Parapoxvirus


7) Butcher's warts are caused by which human papilloma virus (HPV) type?

A. 7Correct Choice

B. 2

C. 5

D. 3

E. 13
Butcher's warts are caused by HPV type 7


8) An 18-year old man presents to the Dermatology Clinic with a nontender penile erosion that has
been present for 2 weeks. An indurated border and nontender bilateral inguinal lymphadenopathy
are also noted. What is the most likely diagnosis?


A. Lymphogranuloma venereum

B. Herpes simplex

C. Chancroid

D. Primary syphilisCorrect Choice

E. HIV
A painless chancre with an indurated border is characteristic of primary syphilis. Associated painless
lymphadenopathy ("buboes") is also a common feature



                                                  2
9) What is the causative agent of this parasitic eruption which also causes patchy pulmonary
infiltrate and eosinophilia?


A. Filariasis

B. Dracunculiasis

C. Larva migransCorrect Choice

D. Larva currens

E. Gnathostomiasis
Larva migrans is caused by Ancylostoma braziliense, a hookworm of cats and dogs. It also
associated with Loeffler's syndrome, patchy infiltrate of the lungs with eosinophilia. Treatment for
this parasitic infection is ivermectin or albendazole


10) One of your colleagues returned from a vacation to India with fever, vomiting, pleural effusions,
ascites and conjunctival petechiae. She also has a diffuse macular rash with notable areas of
sparing that your attending refers to as “white islands in a sea of red”. What is your diagnosis?


A. Malaria

B. Leishmania

C. Leptospirosis

D. Typhoid

E. Dengue hemorrhagic feverCorrect Choice
Dengue fever is caused by an Arbovirus and transmitted by the mosquito, Aedes aegypti and may
cause Dengue Shock Syndrome and Dengue Hemorrhagic Fever. The infection starts with sudden
onset of high fever, backache, retroorbital pain, bone and joint pain, depression and malaise. The
disease is also called "break-bone fever." One to seven days after onset of fever, rash presents
characteristically starting on the dorsum and hands and feet spreading to limbs and torso. The
eruption may become confluent with small, round islands of sparing, the so called "white islands in
a sea of red." Treatment is generally supportive as no antivirals are effective


11) The treatment of choice for acyclovir-resistant herpes simplex virus infection is:

A. Saquinivir

B. Valcyclovir

C. Indinivir

D. FoscarnetCorrect Choice

E. Famciclovir
Foscarnet directly inhibits viral DNA polymerase (without requiring phosphorylation by TK) and is
therefore effective in acyclovir-resistant HSV infections. Cidofovir can also be used


12) Tricomycosis axillaris is caused by:

A. Burkholderia mallei

B. Corynebacterium tenuis Correct Choice

C. Proteus species




                                                  3
D. Micrococcus sedentarius

E. Corynebacterium minitissimum
Corynebacterium tenuis


13) Which of the following are inconsistent with the diagnosis of staphylococcal scalded skin
syndrome?


A. Prognosis is good in children with low mortality when anitbiotics are administered early

B. Swabs and cultures of fluid filled blisters overwhelmingly grow staphCorrect Choice

C. Initial event is usually a localized staph infection

D. Cell necrosis does not occur in staphylococcal scalded skin syndrome as it does in T.E.N

E. Epidermal changes are produced by exfoliative toxins of staphlococcus
Staphylococcal scalded skin syndrome: Lesions extend far beyond areas of actual staphylococcal
infection, by action of the epidermolytic exotoxin elaborated by the staphylococcus in remote sites.
Usually the staphylococci are present at a distant focus such as the parynx, nose ear, or conjuctiva.
If cultures are taken they should be obtained fromthe mucous membranes because the skin
erythema and desquamation is due to the distant effects of the exfoliative toxin, unlike the sitaution
in bullous impetigo, where S. aureus is present in the lesions. Epidermal changes are produced by
exfoliative toxins of Staphylococcus. Inital event is usually a localized Staph infection. Prognosis is
good in children with low mortality when anitbiotics are administered earyl. Cell necrosis does not
occur in SSS as it does in toxic epidermal necrolysis


14) Which of the following is the vector for Dengue fever?

A. Lutzomyia verrucarum

B. Phlebotomus papatasii

C. Xenopsylla cheopis

D. Aedes aegyptiCorrect Choice

E. Culex mosquito
Dengue fever also known as "break-bone" fever is characterized by sudden high fever, backache,
retro-orbital pain, bone/joint pain, weakness, and malaise. It is caused by an arbovirus (RNA virus)
which is transmitted by Aedes aegypti, a species of mosquito


15) Which virus is most commonly associated with oral hairy leukoplakia?

A. Human papilloma virus

B. Epstein-Barr virusCorrect Choice

C. Parvovirus

D. Pox virus

E. Herpes virus
Oral hairy leukoplakia is an oral mucosal infection most often caused by the Epstein-Barr virus,
which occurs in immunocompromised patients. Clinically, thick, white plaques are noted on the
lateral sides of the tongue, often fissured


16) Which Rickettsial infection has a negative Weil-Felix test?



                                                     4
A. Rickettsialpox (R. akari)Correct Choice

B. Rocky Mountain Spotted Fever (R. rickettsii)

C. Epidemic typhus (R. prowazekii)

D. Mediterranean fever (R. conorii)

E. Endemic typhus (R. typhi)
Rickettsia are short, gram-negative rods which are strict intracellular parasites. The bacteria is
transmitted via tick which much be attached for more than 6 hours. The Weil-Felix test exploits
cross-reactivity between the Proteus vulgaris antigen and RIckettsia


17) A pool worker develops these painful erythematous nodules with ulceration on his fingers and
then his arms. The antibiotic of choice to treat this infection is:


A. Ceftriaxone

B. Trimethoprim-sulfamethoxasole

C. MinocyclineCorrect Choice

D. Dapsone

E. Penicillin
Sporotrichoid spread may be seen in cutaneous leishmaniasis, actinomycosis, atypical mycobacterial
infections, deep fungal infection, and melanoma. Mycobacterium marinum is associated with skin
injury followed by exposure to contaminated water, usually from an aquarium, lake, or pool. The
treatment of choice for M. marinum infections is minocycline


18) A 20-month old child develops a high fever followed 2 days later by a sudden eruption of rose
pink macules and papules with white halos as the fever subsides. What is the most likely diagnosis?


A. Measles

B. Rubella

C. Exanthem Subitum Correct Choice

D. Erythema infectiosum

E. Scarlet fever
The cutaneous features of Exanthem Subitum (Roseola infantum, Sixth Disease)as described in the
question develop as the fever subsides. While the other diagnoses are part of the differential
diagnosis of "fever and rash" in children, only roseola infantum has this characteristic clinical course


19) WHO recommendations for the treatment of multibacillary leprosy include all of the following
EXCEPT:


A. Clofazimine

B. All of these answers are recommended for the treatment of multibacillary leprosy.

C. Rifampin

D. Dapsone

E. ClarithromycinCorrect Choice
Clarithromycin is not part of the WHO recommendations for the treatment of multibacillary leprosy




                                                    5
20) The most likely cause of the childhood infection shown is:

A. E. coli

B. Pseudomonas Aeruginosa

C. Group A Streptococcus Correct Choice

D. S. Aureus

E. H. influenza
This is an example of perianal streptococcal “cellulitis.” The causative organism is Group A
streptococcus


21) In addition to Kaposi’s sarcoma, HHV-8 infection is also the causative agent in:

A. Pyogenic granuloma

B. Pityriasis rosea

C. Angiosarcoma

D. Primary effusion lymphomaCorrect Choice

E. Bacillary angiomatosis
Human herpes virus 8 is a double stranded DNA virus. It is thought to be pathogenic in Kaposi’s
sarcoma, primary effusion lymphoma, and Castleman’s disease. Primary effusion lymphoma is a
rare B-cell lymphoma seen predominantly in patients with AIDS


22) The nonvenereal treponematosis shown is caused by which of the following organisms?

A. T. pertenue

B. T. pallidum (subsp. endemicum)

C. B. duttonii

D. T. carateum Correct Choice

E. T. pallidum
The depigmented patches on the hands shown are characteristic of the late (tertiary) stage of pinta.
The causative organism is T. carateum


23) A 52 year-old butcher presents with a tender lesion on his left hand as shown. The treatment of
choice for this condition is:


A. Chloramphenicol

B. Ciprofloxacin

C. Tetracycline

D. Penicillin Correct Choice

E. Azithromycin
The clinical description and lesion shown in the image suggest a diagnosis of erysipeloid, caused by
Erysipelothrix rhusiopathiae. The treatment of choice is penicillin


24) The treatment of choice of cat-scratch disease is:



                                                  6
A. Itraconazole

B. Mebendazole

C. Ceftriaxone

D. Trimethoprim-sulfamethoxazole

E. ErythromycinCorrect Choice
Cat-scratch disease is caused by Bartonella henselae, which is spread among cats by fleas. The
hallmark of the disease is unilateral and regional lymphadenopathy. The treatment of choice for cat-
scratch disease is erythromycin


25) Pretibial fever is caused by:

A. Listeriosis

B. Rickettsiae spp.

C. LeptospirosisCorrect Choice

D. Rickettsiae spp.

E. Ehlichiosis
Leptospira autumnalis causes pretibial fever (“Fort Bragg fever” or anicteric leptospirosis), an acute
exanthematous infectious erythema, generally most prominent on the shins. Leptospira interrogans,
serotype icterohaemorrhagiae, is the most common cause of Weil’s disease or icteric leptospirosis


26) Guarneri bodies are associated with:

A. Smallpox Correct Choice

B. Anthrax

C. Measles

D. Orf

E. CMV
Guarnieri’s bodies are cytoplasmic eosinophillic inclusions found on light microscopy


27) Petechial lesions at the junction of the hard and soft palate are characteristically seen in which
disease?


A. Kawasaki’s disease

B. Infectious mononucleosisCorrect Choice

C. Scarlet fever

D. Rocky Mountain spotted Fever

E. Rickettsialpox infection
Forsheimer spots are distinctive pinhead size petechiae found at the junction of the hard and soft
palate. They may be seen in infectious mononucleosis


28) Congenital Varicella Syndrome occurs after maternal varicella infection during which stage of
pregnancy?


A. Third trimester



                                                   7
B. First 20 weeks Correct Choice

C. 20-24 weeks

D. 5 days before and 2 days after delivery

E. None of these answers are correct
Congenital Varicella Syndrome occurs after maternal varicella-zoster virus infection early in
pregnancy (up to 20 weeks gestation)


29) Of the choices listed, which antiobiotic is the best to treat a 7-year old child with erythema
migrans?


A. Tetracycline

B. Azithromycin

C. AmoxicillinCorrect Choice

D. Doxycycline

E. Rifampin
Doxycycline is the usual treatment for erythema migrans in adults in the absence of neurologic or
cardiovascular complications. In children less than 8, all tetracyclines are relatively contraindicated
because of they can cause tooth discoloration. Therefore amoxicillin should be used as the first line
therapy for children less than 8. Note: Changes have been made to the treatment of RMSF. Children
of any age should be treated with doxycycline for that disease


30) A 48 year-old woman presents to the Emergency Room with a 2 day history of fever and a well
demarcated erythematous plaque on her right cheek. What is the treatment of choice?


A. Potent topical corticosteroid

B. Fluconazole

C. Intravenous penicillinCorrect Choice

D. Prednisone

E. Cephelexin
The most likely diagnosis is Erysipelas which is caused by beta-hemolytic group A streptococcus.
The treatment of choice is intravenous penicillin


31) Herpes simplex virus infection and this condition has been linked to which HLA type?

A. HLA B15Correct Choice

B. HLA B51

C. HLA DR3

D. HLA Cw6

E. HLA B27
HSV related erythema multiforme has been associated with an increased frequency of HLA-B15


32) Milker’s nodule is caused by infection with which of the following viruses?

A. Orf virus




                                                   8
B. Human herpesvirus-8 (HHV-8)

C. Paravaccinia virusCorrect Choice

D. Human herpesvirus-6 (HHV-6)

E. Vaccinia virus
Milker’s nodule is caused by paravaccinia virus, a poxvirus of the genus Parapoxvirus. It is
transmitted to humans from infected cows


33) Which porphyrin is responsible for the fluorescence under Wood's lamp examination of
erythrasma?


A. Protoporphyrin IX

B. Porphobilinogen

C. Coproporphyrin IIICorrect Choice

D. Coproporphyrinogen

E. Uroporphobilinogen
Erythrasma is caused by corynebacterium minutissimum. The presence of coprophyrinogen III
created by the bacteria is responsible for the characteristic coral-red fluorescence under Wood's
lamp


34) Epithelioma cuniculatum is an HPV-associated verrucous carcinoma involving the:

A. Trunk

B. Scalp

C. Penis

D. Oral mucosa

E. SoleCorrect Choice
Epithelioma cuniculatum is a form of verrucous carcinoma involving the sole


35) A 28 year-old man presents with fever, chills, arthralgias and the cutaneous lesions shown. The
most likely diagnosis is:


A. Gonoccemia Correct Choice

B. Rickettsialpox

C. Primary herpes simplex infection

D. Anthrax

E. Purpura fulminans
The image shows sparsely distributed hemorrhagic vesiculopustules with erythematous bases on an
acral surface. These features are characteristic of gonococcemia


36) Blueberry Muffin Baby can be a feature of all of the following EXCEPT:

A. Rhabdomyosarcoma

B. Langerhans’ cell histiocytosis




                                                  9
C. TORCH

D. Hemolytic disease of the newborn

E. Klippel-Trenaunay-Weber syndrome Correct Choice
Blueberry muffin lesions can be seen in the setting of prenatal infections (e.g. TORCH), severe
anemia (e.g. Hemolytic Disease of the newborn) and neoplastic diseases (e.g. rhabdomyosarcoma).
It is not associated with the Klippel-Trenaunay-Weber syndrome


37) The treatment of choice for Oroya Fever is:

A. ChloramphenicolCorrect Choice

B. Erythromycin

C. Penicillin

D. Minocycline

E. Doxycycline
The treatment of choice for Oroya Fever is chloramphenicol
because of frequent superinfection with
Salmonella


38) A 10 year-old with a 3-day history of fever and headache presents to the clinic with multiple
erythematous papulopustules on his extremities, many of which have central eschars. The most
likely diagnosis is:


A. Erlichiosis

B. RickettsialpoxCorrect Choice

C. Meningococcemia

D. Lyme disease

E. Rocky mountain spotted fever
Rickettsialpox is an acute febrile illness caused by the bacteria Rickettsia akari, a member of the
spotted fever group. R. akari is transmitted by the house mouse mite, Liponyssus sangineus.
Clinically, patients have an eschar at the inoculation site with a subsequent papulovesicular rash,
fevers, headache, chills, diaphoresis, myalgia, and anorexia


39) A 35 year-old woman presents with a painful vaginal erosion and tender left-sided inguinal
lymphadenopathy. Gram stain reveals clusters of coccobacilli in a “school of fish” pattern. What is
the most likely diagnosis?


A. Granuloma inguinale

B. ChancroidCorrect Choice

C. Primary syphilis

D. Secondary syphilis

E. Lymphogranuloma venereum
Chancroid classically presents with a soft, tender chancre with ragged edges and unilateral, tender
inguinal adenopathy (bubo). A “school of fish” pattern on Gram or Giemsa stain is diagnostic


40) All of the following are features of scarlet fever except:



                                                  10
A. S. aureus infectionCorrect Choice

B. Pastia’s lines

C. Pharyngitis

D. Circumoral pallor

E. Sandpaper-like texture
Scarlet fever is primarily a disease of children with most cases occurring between the ages of 1 and
10 years of age. Streptococcus pyogenes is the causative organism. It produces the streptococcus
pyrogenic exotoxin which elicits the cutaneous manifestations by enhancing delayed type
hypersensitivity to streptococcal products. Clinical findings include fever, sore throat, headache,
chills, sandpaper rash on the trunks, strawberry tongue, and Pastia’s lines (linear petechial streaks
found in flexural locations


41) Hebra nose deformity is characteristic of infection with which organism?

A. Serratia marcesens

B. Haemophilus influenzae

C. Streptococcus pyogenes

D. E. coli

E. Klebsiella pneumoniaeCorrect Choice
Hebra nose deformity is seen in Rhinoscleroma which is caused by Klebsiella pneumoniae


42) Pediculus humanus corporis can transmit:

A. Scrub typhus

B. Oriental typhus

C. Epidemic typhusCorrect Choice

D. Endemic typhus

E. Murine typhus
The body louse or Pediculus humanus corporis transmits Rickettsia prowasekii the organism
responsible for epidemic typhus. Endemic typhus or murine typhus is caused by R. typhi that is
transmited by the rat flea/Xenopsylla cheopis. Scrub typhus is caused by R. tsutsugamushi and is
transmitted by chiggers or trombiculid mite larvae


43) Xenopsylla cheopis transmits:

A. Rickettsia akari

B. Epidemic typhus

C. Scrub typhus

D. Murine typhusCorrect Choice

E. Rickettsia prowazekii
Xenopsylla cheopis along with Ctenocephalides felis are fleas that transmit R. typhi, the organism
responsible for murine or endemic typhus. The body louse or Pediculus humanus corporis transmits
Rickettsia prowasekii the organism responsible for epidemic typhus. Scrub typhus is caused by R.
tsutsugamushi and is transmitted by chiggers or trombiculid mite larvae




                                                 11
44) A 4 year-old boy is diagnosed with Staphylococcal Scalded-Skin Syndrome. Vesiculation in this
disease is secondary to exfoliative toxins binding what target protein?


A. Desmoglein 1Correct Choice

B. Bullous Pemphigoid antigen 1

C. Bullous pemphigoid antigen 2

D. Desmoglein 3

E. Plectin
Staphylococcus Scalded-Skin Syndrome (SSSS) is caused by Staphylococcus exfoliative toxins A
and B binding desmoglein A. This results in the intraepidermal split at the granular layer seen in this
illness. Targeting of desmoglein 3 would result in a suprabasilar split. Targetting of plectin, BPAG1,
or BPAG2 would result in a subepidermal split. This is not seen in SSSS


45) Streptococcus Iniae has been shown to cause:

A. Perianal dermatitis in neonates

B. Bullous impetigo

C. Perineal erysipelas in postpartum women

D. Necrotizing fasciitis

E. Hand cellulitis in fish handlersCorrect Choice
Streptococcus iniae has been demonstrated to cause hand cellulitis from puncture wounds sustained
form the dorsal fin, fish bone or knife of usually a tilapia. Treatment with PCN is curative. Group A
Streptococci are the most common cause of perianal dermatitis. Many different bacteria have been
implicated in necrotizing fasciitis. Bullous impetigo is most frequently caused by phage type 71 S.
aureus or a related group 2 phage type. Group B streptococcus is most often responsible for
perineal erysipelas in postpartum women.


46) What is the most common site of infection from Streptococcus Iniae?

A. Face

B. HandCorrect Choice

C. Lower leg

D. Foot

E. Nails
Streptococcus iniae most commonly causes bacteremic cellulitis of the hand in persons who have
recently handled fresh fish. S. iniae is a fish pathogen that causes sporadic infection in tilapia,
yellowtail, rainbow trout, and coho salmon. Treatment of choice is with penicillin antibiotics


47) The vector of New World Leishmaniasis is the:

A. Deer fly (Chrysops spp.)

B. Tsteste fly

C. Phlebotomus sandfly

D. Lutzomyia sandflyCorrect Choice

E. Simulium black fly



                                                    12
The sandfly belonging to the genus Lutzomyia is the vector of New World Leishmaniasis. Sandflies
of the genus Phlebotomus are the vectors of Old World Leishmaniasis


48) How long is the life cycle for the mite that causes this infestation?

A. 1 day

B. 1 monthCorrect Choice

C. 6 months

D. 1 year

E. 1 week
Scabies is caused by the mite Sarcoptes scabiei var. hominis. The life cycle of the mite is 30 days. A
female mite will lay 60-90 eggs during her life


49) Herpes-associated erythema mutliforme is most commonly associated with which HLA type?

A. HLA DR2

B. HLA DR3

C. HLA B15Correct Choice

D. HLA B8

E. HLA B27
Recurrent erythema multiforme (EM minor) is usually caused by recurrent herpes simplex, most
commonly HSV-1 orolabial disease. This is more correctly now called herpes-associated erythema
multiforme(HAEM) and has been more commonly associated with people with HLA type B15


50) All of the following are features of the Ramsay Hunt Syndrome EXCEPT:

A. Infection of the geniculate ganglion

B. Vesicles on the external ear

C. Tinnitus

D. Facial paresis

E. Herpes simplex infectionCorrect Choice
Ramsay Hunt syndrome results from varicella zoster virus infection of the geniculate ganglion of the
seventh cranial nerve (CN VII). It is characterized by vesicles on the external ear or ear canal,
tinnitus and/or other auditory symptoms, and ipsilateral facial paresis


51) Exanthem Subitum is caused by which of the following?

A. Group A Steptococcus

B. Coxsackie virus

C. Human Herpes Virus-6 (HHV-6) Correct Choice

D. Parvovirus B19

E. Epstein-Barr Virus
Exanthem Subitum (Roseola Infantum, Sixth Disease) is caused by human herpesvirus 6, and 7
(HHV-6, HHV-7




                                                  13
52) Carrion's disease is characterized by fevers, headaches and arthralgias and is accompanied by
severe hemolytic anemia. Superinfection with which organism is the most frequent cause of death?


A. Shigella

B. Bartonella henselae

C. SalmonellaCorrect Choice

D. Bartonella bacilliformis

E. Bartonella Quintana
Carrion's disease (Oroya fever) is characterized by fevers, headaches and arthralgias and is
accompanied by severe hemolytic anemia. Superinfection with Salmonella is the most frequent
cause of death. Protection from sandfly bites is all-important


53) Dracunculiasis is typically acquired from:

A. Soil

B. Salt water

C. Black flies

D. Drinking water Correct Choice

E. Chiggers
Dracunculiasis, which is caused by Dracunculus medinensis, is acquired by ingestion of copepod-
infested water


54) Which subtype of the human papilloma virus is most likely to have caused this infection?

A. HPV, type 3

B. HPV, type 6Correct Choice

C. HPV, type 1

D. HPV, type 5

E. HPV, type 2
The human papilloma virus is a member of the Papovavirus family, a double-stranded DNA virus.
HPV, type 6 has been implicated in giant conduloma of Buschke and Lowenstein and anogenital
condyloma


55) Which of the following diseases is caused by a virus?

A. Boutonneuse spotted fever

B. Colorado tick feverCorrect Choice

C. Q fever

D. Rocky Mountain spotted fever

E. Human monicytic ehrlichiosis
Rocky Mountain spotted fever is caused by Rickettsia rickettsii. Human monicytic ehrlichiosis is
caused by Ehrlichia chaffeensis. Colorado tick fever is caused by Reoviiridae. Boutonneuse fever is
caused by Rickettsia conorii. Q fever is caused by Coxiella burnetti




                                                 14
56) Epidermodysplasia verruciformis is caused by which human papilloma virus (HPV) types?

A. 3 and 10

B. 6 and 11

C. 5, 8, and 9Correct Choice

D. 13 and 32

E. 16 and 18
Epidermodysplasia verruciformis is caused by HPV types 5, 8, 9, 12, 14, 15, 17, 19-26


57) Verruga peruana is transmitted by:

A. Fecal oral contact

B. Blackfly

C. SandflyCorrect Choice

D. Tick

E. Fecal fecal contact
Verruga peruana, a disease endemic to Peru and a few neighboring countries, is caused by
Bartonella bacilliformis. It is transmitted by the sandfly, Lutzomyia verrucarum. It is preceded by an
acute febrile stage called Oroyo fever or Carrion’s disease. The treatment of choice is
chloramphenicol


58) Each of the following is true regarding leishmaniasis except:

A. Arthropod vector is the sand fly

B. Mazzotti’s test is diagnosticCorrect Choice

C. Cutaneous disease is the most common form

D. Pentavalent antimony used for visceral disease

E. Espundia seen mucocutaneous disease
Leishmaniasis is a parasitic infection born by a sandfly vector. No treatment is necessary for
cutaneous leishmaniasis, pentavalent antimony for visceral leishmaniasis. Mazzotti reaction is used
to test for onchocerciasis in which a single dose of dimethycarbamazine is given for reaction


59) What is the principal vector of Lyme Disease in the Northeastern U.S.?

A. Lxodes ricinus

B. Dermacentor variabilis

C. Lxodes damminiCorrect Choice

D. Ambylomma americanum

E. Soft-bodied ticks (Ornithodoros)
The proncipal vector of Lyme Disease in the Northeastern U.S. is Ixodes dammini (Ixodes scapularis


60) Which of the following mycobacterial organisms is classified as a photochromogen?



                                                 15
A. M. tuberculosis

B. M. grodonae

C. M. KansasiiCorrect Choice

D. M. ulcerans

E. M. fortuitum
M. Kansasii, M. marinum, and M. simiae are photochromogens. They form pigment when exposed to
light


61) The vector of Trench Fever is the:

A. Cat flea (Ctenocephalides felis)

B. Human body louse (Pediculus humanus corporis)Correct Choice

C. Trombiculid mite

D. Sandfly (Phlebotamus perniciosus)

E. Rat flea (Xenopsylla cheopis)
The human body louse (Pediculus humanus corporis)is the vector of Trench Fever, Epidemic Typhus,
and Relapsing Fever


62) Pediculus humanus var. corporis (human body louse) is the vector in which of the following
diseases:


A. Q fever (C. burnetii)

B. Endemic typhus (R. typhus)

C. Rickettsialpox (R. akari)

D. Epidemic typhus (R. prowazekii)Correct Choice

E. Rocky Mountain Spotted Fever (R. rickettsii)
The human body louse is the implicated Trench fever, epidemic typhus, and relapsing fever. Trench
fever usually affects alcoholic men and is manifested by fevers. Treatment is with ceftriaxone,
erythromycin, or doxycycline. Epidemic typhus is manifested by fevers, chills, malaise, and a pink
macular eruption beginning in the axilla and trunk. Treatment is with tetracycline or
chloramphenicol. Relapsing fever is manifested by paroxysmal fevers, headache, lymphocytoma,
and erythematous/petechial macules on trunk/extremities. Treatment is with doxycycline


63) A 27 year-old man with a 1-month history of bilateral inguinal lymphadenopathy and a positive
“groove sign.” What is the most likely diagnosis?


A. Gonorrhea

B. Lymphogranuloma venereumCorrect Choice

C. Syphilis

D. Chancroid

E. Herpes simplex
Inguinal adenopathy with fluctuant, tender nodes above and below Poupart’s ligament - referred to
as “groove sign” - is a characteristic clinical feature of lymphogranuloma venereum




                                                    16
64) The most common location of herpes gladiatorum is the:

A. FaceCorrect Choice

B. Anterior thigh

C. Groin

D. Hand

E. Chest
The face is the most common location for Herpes Gladiatorum


65) Which of the following is the vector of lymphatic filariasis caused by Wuchereria bancrofti?

A. Black fly

B. Deerfly

C. Mosquito Correct Choice

D. Botfly

E. Tsteste fly
Lymphatic filariasis is spread by mosquitoes belonging to genera Aedes, Anopheles, Culex, or
Mansonia


66) What is the classic CXR finding of a patient with inhalational anthrax?

A. Pleural effusions

B. Increased interstitial markings

C. Widened mediastinumCorrect Choice

D. Alveolar infiltrates

E. Normal x-ray
Inhalation anthrax is the most lethal form of anthrax. Other forms include cutaneous anthrax and GI
anthrax. The classic radiographic finding is a widened mediastinum


67) What is the vector for this nematodal infection?

A. Tsetse fly (Glossine morsitans)

B. Mango fly (Chrysops)Correct Choice

C. Black fly (Simulium)

D. Water flea (Cyclops)

E. Reduviid bug
Loiasis is caused by the nematode loa loa. It is transmitted by the mango fly (Chrysops dimidia or
CHrysops silacea). It may manifest with painful, localized subcutaneous nonpitting edema called
calabar swellings. Conjunctival migrations are also common


68) A goatherder develops this painless pustule that subsequently forms a black eschar. What
causes the edema to form in this lesion?




                                                 17
A. Inhibition of phagocytosis

B. C1 esterase inhibitor

C. Increased levels of cAMPCorrect Choice

D. Release of TNF-alpha

E. Release of IL-1beta
Anthrax is caused by Bacillus anthracis, a gram positive spore-forming rod. Anthrax is primarily
seen in individuals who are in contact with wild or domestic animals. Recent cases have been linked
with bioterrorism. B. anthracis produces edema toxin and lethal toxin. Edema toxin is comprised of
edema factor and protective antigen. Edema factor is a calmodulin dependent adenyl cyclase. The
increased cAMP induces the gelatinous edema of anthrax skin lesions


69) Which human papillo viurs (HPV) type is implicated in papillomatosis cutis carcinoides di
Gottron?


A. 7

B. 11Correct Choice

C. 60

D. 18

E. 57
Papillomatosis cutis carcinoides di Gottron, also know as Gottron's tumor, is a verrucous carcinoma
of the skin. In 1948, Ackerman first described verrucous carcinoma (VC), a low-grade tumor that
generally is considered a clinicopathologic variant of squamous cell carcinoma.
Verrucous carcinomas are caused by HPV types 6 and 11. When they occur on the feet they are
called epithelioma caniculatum. When they occur on the genitals they are refered to as Buschke-
Lowenstein tumors. When they occur in the mouth they are called Ackerman's tumors or oral florid
papillomatosis


70) A 48 year-old man develops headache, myalgias, and high fever 5 days after a hunting trip. On
examination he has a tender ulcer with raised margins and an eschar on his index finger, as well as
tender axillary lymphadenopathy. The most likely diagnosis is:


A. Orf

B. Anthrax

C. Tularemia Correct Choice

D. Glanders

E. Erysipeloid
Tularemia, which is caused by Francisella tularensis, occurs after exposure to infected animals,
including, rabbits, foxes, and squirrels. The clinical description is that of the ulceroglandular form,
which is the most common presentation


71) Which exotoxin(s) are involved in mediating the effects of Bacillus anthracis - the causative
organism for anthrax?


A. Lethal Toxin

B. Edema Toxin and Lethal ToxinCorrect Choice

C. Exotoxin B




                                                   18
D. Edema Toxin

E. Alpha toxin
B. anthracis has 2 exotoxins: edema toxin and lethal toxin, each comprised of a pair of
noncovalently linked
proteins:
– Edema toxin = Edema Factor (EF) + Protective Antigen (PA)
– Lethal toxin = Lethal Factor (LF) + PA
– EF causes gelatinous edema of anthrax skin lesions by inducing an increase in cyclic adenosine
monophosphate (cAMP) levels
– LF causes shock and death in disseminated anthrax via the release of tumor necrosis factor-alpha
(TNF-a)and interleukin-1b (IL-1b)
– PA is responsible for entry of exotoxins into the cell by receptor-mediated endocytosis


72) A 43 year-old man with uncontrolled HIV disease presents with yellowish, red papules, draining
sinuses and ulcers perianally and periurethrally. Which histopathological or immunohistochemical
stain may aid in diagnosis?


A. Von Gieson (EVG)

B. CD3

C. Steiner

D. CD20

E. Von KossaCorrect Choice
The most likely diagnosis is malakoplakia. Malakoplakia, an infection usually attributed to S. aureus,
P. aeruginosa, or E. coli is characterized histopathologically by Michelis-Gutmann bodies. These are
foamy eosinophilic macrophages containing calcified, laminated, intracytoplasmic bodies. They can
be highlighted by a calcium stain such as von Kossa. CD3 and CD20 are lymphocyte markers, von
Gieson is an elastic tissue stain, and Steiner stain is used to highlight spirochetes. These would not
aid in the diagnosis of malakoplakia


73) A 42 year-old woman presents with a large, vegetating ulcer involving her left labia majora and
groin for over 1 year. A Giemsa’s stained touch preparation reveals bipolar, safety pin-shaped
intracytoplasmic inclusions. What is the most likely diagnosis?


A. Chancroid

B. Lymphogranuloma venereum

C. Gonorrhea

D. Primary herpes simplex

E. Granuloma InguinaleCorrect Choice
Bipolar, safety pin-shaped intracytoplasmic inclusions on Giemsa stain - known as Donovan bodies -
are diagnostic of granuloma inguinale


74) The leading infectious cause of deafness and mental retardation in the U.S. is:

A. Congenital herpes simplex

B. Congenital syphilis

C. Congenital Cytomegalovirus (CMV)Correct Choice

D. Congenital varicella




                                                 19
E. Congenital Rubella
Congenital CMV is the leading infectious cause of deafness and mental retardation in the U.S.


75) Which human papillomavirus type is associated with giant condylomata of Bushke and
Lowenstein (Bushke-Lowenstein tumor)?


A. 2

B. 31

C. 16

D. 1

E. 6 Correct Choice
The Bushke-Lowenstein tumor is caused by HPV-6 and 11


76) Each of the following species may be involved in necrotizing fasciitis except:

A. MycobacteriumCorrect Choice

B. Pseudomonas

C. Bacteroides

D. Streptococcus

E. Clostridium
Necrotizing fasciitis is a rapidly advancing acute necrotizing infection that may follow trauma,
surgery, or occur spontaneously. It is associated with systemic toxicity and high mortality rate.
Clinically, erythema, edema, and edema progresses to dusky cyanosis, blistering and necrosis.
Many virulent bacteria have been culturesd including Pseudomonas, Bacteroides, streptococci,
staphylococcus, enterococci, and clostridium. Both aerobic and anaerobic cultures should be taken.
Mycobacterium is not associated with necrotizing fasciitis


77) Bacillary angiomatosis is caused by which of the following organisms?

A. B. quintana

B. All of these answers are correct

C. B. quintana and B. henselaeCorrect Choice

D. B. bacilliformis

E. B. henselae
B. quintana or B. henselae are both causative organisms for bacillary angiomatosis


78) A 24 year-old man from New York City develops fever, chills, malaise, and a severe headache.
His apartment has a mouse problem. On examination, multiple, discrete papulovesicles and a single
eschar are found. What is the most likely diagnosis?


A. RickettsialpoxCorrect Choice

B. Rocky Mountain Spotted Fever

C. Q Fever

D. Meningococcemia




                                                 20
E. Ehrlichiosis
The clinical description is that of rickettsialpox, which is caused by R. akari and spread by
liponyssoides sanguineus - the house mouse mite


79) Nonmenstrual cases of the staphylococcal toxic shock syndrome are mediated by which of the
following toxins?


A. Epidermolytic toxins A and B

B. Edema toxin

C. Pyrogenic exotoxins A, B, C

D. Exfoliative toxins A and B

E. Enterotoxins B and C Correct Choice
Toxic shock syndrome (TSS) is mediated by Enterotoxins B and C (50% of nonmenstrual cases) and
TSS toxin 1 (TSST-1), which is the leading cause of menstrual cases


80) A 27 year-old HIV positive presents with this pruritic eruption? What is the characteristic cell
type found on histopathology?


A. Plasma Cell

B. Lymphocyte

C. Neutrophil

D. EosinophilCorrect Choice

E. Neutrophil
Eosinophilic pustular folliculitis is a pruritic disorder that may be seen in HIV positive patients.
Typically, the CD4 count is < 300. In patients with HIV, pruritus tends to improve after institution of
anti-retroviral therapy


81) Which of the following statements best describes the pathogenesis of scrofuloderma?

A. Autoinoculation of M. tuberculosis from advanced pulmonary tuberculosis

B. Hematogenous spread of M. tuberculosis from a distant site of infection

C. Primary incoculation of M. tuberculosis in a sensitized host

D. An cutaneous manifestation of military tuberculosis

E. Contiguous spread from an underlying focus of tuberculous lymphadenitis Correct Choice
Scrofuloderma is the result of contiguous spread onto skin from an underlying focus of tuberculous
infection. This typically occurs in a sensitized host with low immunity


82) Focal epithelial hyperplasia (Heck’s Disease) is associated with which of the following human
papillomavirus types?


A. 7

B. 13 Correct Choice

C. 10

D. 4




                                                  21
E. 3
Focal epithelial hyperplasia is caused by HPV-13 and 32


83) Winterbottom’s sign is a characteristic feature of which of the following diseases?

A. Onchocerciasis

B. Leishmaniasis

C. Loaiasis

D. Strongyloidiasis

E. African Trypanosomiasis Correct Choice
Winterbottom’s sign (posterior cervical lymphadenopathy) is a clinical feature of African
trypanosomiasis. Romaña’s sign (eyelid edema and conjunctivitis at site of incoculation) is a
diagnostic finding in American trypanosomiasis (Chagas’ disease)


84) A young patient presents to you after participating in a bar brawl a few nights before with a
painful swollen erythematous right hand. The most likely organism is:


A. Staphylococus aureus

B. Bacteriodes fragilis

C. Eikenella corrodensCorrect Choice

D. Pasturella multocida

E. Streptococcus pyogenes
Eikenella corrodens is a member of normal oral flora, that is commonly the cause of human bite and
fist fight infections. Pasturella multocida is a common organism found in dog and cat bites. Staph
and Strep could be the right answers for a cellulitis, but with this history are not the correct choices.
Baceriodes fragilis is found in normal bowel flora


85) Hutchinson’s teeth are a feature of which stage of syphilis?

A. Primary

B. Secondary

C. Tertiary

D. Early congenital

E. Late congenital Correct Choice
Hutchinson’s teeth (widely-spaced, peg-shaped upper incisors) are a seen in Late Congenital
Syphilis (> 2 years of age


86) The causative organism in Whitmore disease is which of the following?

A. Vibrio vulnificus

B. Pseudomonas aeruginosa

C. Streptobacillus moniliformis

D. Burkholderia pseudomalleiCorrect Choice

E. Klebsiella pneumoniae



                                                   22
Whitmore disease also known as Meloidosis is characterized by pulmonary disease, septicemia, and
miliary abscesses. It is caused by Burkholderia pseudomallei


87) All of the following are true regarding tularemia except:

A. Potential for use as a biologic weapon

B. Ticks serve as a reservoir

C. No longer one of the CDC's list of reportable diseasesCorrect Choice

D. Presents as an acute febrile illness

E. Caused by the gram negative Francisella tularensis
Tularemia is a zoonotic disease caused by the gram-negative coccobacillus Francisella tularensis. It
is transmitted by arthropods (particularly tick bites) or from contact with infected mammals (usually
rabbits). It characteristically presents as an acute febrile illness with other variable clinical
manifestations depending on the route of inoculation. Possible symptoms could include an ulcer at
the site of inoculation, pharyngitis, ocular lesions, lymphadenopathy, and pneumonia. Diagnosis can
be made by culture or a fourfold titer change. Because of its potential use as a biologic agent,
tularemia is once again a reportable disease


88) All of the following are potential causes of a false positive RPR except:

A. Pregnancy

B. Malaria

C. Malignant MelanomaCorrect Choice

D. Systemic Lupus Erythematosus (SLE)

E. Lepromatous Leprosy
Non-treponemal tests for syphilis measure antibodies against phospholipids released from
treponemes and damaged host cells. False positive non-treponemal tests can occur in the setting of
pregnancy, autoimmune diseases, other spirochete infections, and infectious diseases such as
leprosy and malaria. It does not occur in cases of melanoma


89) Coxsackievirus A16 is implicated in the pathogenesis of which of the following dieases?

A. Fifth disease

B. Exanthem subitum

C. Papular purpuric “gloves and socks” syndrome

D. Hand-Foot-and-Mouth disease Correct Choice

E. Erythema infectiousum
Hand-Foot-and-Mouth disease is caused by coxsackievirus A16. Group A coxsackievirus infection is
also associated with herpangina


90) The most common bacterial cause of purpura fulminans is:

A. Group A streptococcus Correct Choice

B. E. coli

C. P. aeruginosa




                                                  23
D. S. aureus

E. H. influenzae
Group A streptococcus is the leading bacterial cause of purpura fulminans


91) Which of the following is the causative organism of Tularemia?

A. Orientia tsutsugamushi

B. Klebsiella pneumoniae

C. Burkholderia mallei

D. Francisella tularensisCorrect Choice

E. Streptobacillus moniliformis
Tularemia is caused by infection with Francisella tularensis - most commonly acquired after contact
with infected rabbits (e.g. hunters


92) Roseola infantum is caused by which virus?

A. Pox virus

B. Coxsackie virus A16

C. Parvovirus B19

D. Human herpes virus 6Correct Choice

E. Epstein-Barr virus
Roseola infantum may be cause be either human herpes virus types 6 or 7. It is the sixth of the
traditional exanthems of childhood. A diffuse, maculopapular eruption is typically preceded by a
prodrome of fever and either respiratory or gastrointestinal symptoms. The infection usually occurs
in the spring and the fall and nearly all children are seropositive for the virus by the age of 4


93) Foamy macrophages containing Klebsiella pneumoniae are called:

A. Mikulicz's cellsCorrect Choice

B. Virchow cells

C. Michaelis-Gutman cells

D. Russell cells

E. Hansemann's cells
Mikulicz’s cells are foamy macrophages found in Rhinoscleroma – a chronic, inflammatory,
granulomatous disease of the upper respiratory tract. Virchow cells are found in Hansen’s disease.
Hansemann cells and Michaelis-Gutman bodies are found in Malakoplakia


94) The diagnosis of leishmaniasis can be confirmed by which of the following techniques?

A. Culture in Sabourad's agar

B. Warthin-Starry stain

C. Culture in Michel's medium

D. Weil-Felix test




                                                 24
E. Culture in Novy-MacNeal-Nicolle (NNN) mediumCorrect Choice
Confirmation of infection with Leishmaniasis is performed by culture in Novy-MacNeal-Nicolle (NNN)
medium


95) What is the most common cause of focal epithelial hyperplasia?

A. Pachyonychia Congenita

B. human immunodeficiency virus(HIV)

C. Dyskeratosis Congenita

D. human papilloma virus(HPV)Correct Choice

E. Ebstein Bar virus(EBV)
Focal epithelia hyperplasia, also known as Heck's disease, is caused by HPV types 13, 32 and 57. It
is most commonly found on the lower lip but also on the buccal mucosa, gums and tongue. It is
mainly a disease of native Americans and Greenlander Eskimos. EBV causes an oral hairy
leukoplakia in patients with AIDS. Pachyonychia Congenita patients commonly develop a benign oral
leukoplakia, while Dykeratosis Congenita patients develop a premalignant oral leukoplakia


96) Which form of cutaneous tuberculosis is associated with the highest level of immunity?

A. Tuberculosis gumma

B. Tuberculosis verrucosa cutis

C. Lupus vulgarisCorrect Choice

D. Miliary tuberculosis

E. Tuberculosis chancre
Lupus vulgaris is an chronic, progressive form which usually occurs around nose, neck and earlobes,
notable for gelatinous "apple-jelly" hue. Tuberculosis chancre and tuberculosis verrucosa cutis cutis
are primary inoculations of tuberculosis infection. Tuberculosis gumma and miliary tuberculosis are
associated with lowered states of immunity


97) Which of the following is the causative organism of cat-scratch disease?

A. B. quintana

B. B. burgdorferi

C. B. bacilliformis

D. B. henselae Correct Choice

E. B. duttonii
B. henselae is the cause of cat-scratch disease. Humans are infected by a cat bite or scratch, while
the cat flea is responsible for cat to cat transmission


98) Herpangina is caused by which of the following?

A. Parvovirus B19

B. Group A coxsackievirusCorrect Choice

C. Human Herpesvirus 6 (HHV-7)




                                                 25
D. Epstein Barr Virus

E. Human Herpesvirus 6 (HHV-6)
Herpangina is caused by Group A coxsackievirus


99) A patient on the bone marrow transplant service has a fever, neutropenia, hemorrhagic bullae
and erythematous nodules with dusky gray centers. The organism most likely to be responsible is:


A. Streptococcus pyogenes

B. Pseudomonas aeruginosaCorrect Choice

C. Mycobacterium tuberculosis

D. Staphylococcus aureus

E. Candida albicans
Ecthyma gangrenosum is an infection of critically ill or immunocompromised individuals by
Pseudomonas aeruginosa. Clinically, patients develop opalescent, tense vesicles or pustules
surrounded by a narrow pink to violaceous halo. The lesions rapidly become hemorrhagic, then
violaceous and necrotic leaving ulcers. The most common location is on the buttocks and lower
extremities. Treatment is with aminoglycosides


100) What is the treatment of choice in a penicillin allergic individual?

A. Rifampiin

B. Penicillin

C. DoxcycylineCorrect Choice

D. Ceftriaxone

E. Clindamycin
Secondary syphillis is caused by the spirochete, T. pallidum. Penicillin is the treatment of choice for
secondary syphillis. In penicillin allergic individuals, doxycyline may be used


101) A patient presents with this anesthestic plaque. Which cytokines are upregulated in this
disease?


A. IL-2Correct Choice

B. All of these cytokines are upregulated

C. IL-5

D. IL-10

E. IL-4
Leprosy is caused by infections to Mycobacterium leprae. It is a acid fast bacilli. In tuberculoid
leprosy, Th1 cytokines (IL-2, IFN-gamma) are upregulated. Clinically, patients have few, well-
circumscribed anesthetic plaques


102) A 43 year-old man presents with suppurative nodules and sinus tracts over the mandible. He
also is noted to have poor oral hygiene. What is the most likely causative organism?


A. Staphylococcus aureus

B. Nocardia brasilensis




                                                  26
C. Streptococcus somaliensis

D. Bartonella Henselae

E. Actinomyces israeliiCorrect Choice
Actinomyces israelii is an anaerobic gram-positive rod which causes chronic suppurative nodules
and sinus tracts with an exudate containing sulfur granules. It most commonly effects the
cervicofacial area, especially near the mandible. The abdomen and thoracic areas can also be
affected. The source of infection is endogenous and patients with poor oral hygiene, penetrating
foreign bodies, and recent dental procedures are at increased risk. Treatment of choice is penicillin


103) A 32 year-old woman presents with meningitis and palpable purpura. A diagnosis of
meningococcemia is confirmed via culture of cerebrospinal fluid. Which complement components are
most likely to be deficient in this patient?


A. C3

B. C2

C. C4

D. C5Correct Choice

E. C1
Patients with meningococcemia often demonstrate deficiencies of late complement components
C5-9.


104) A 62 year-old man presents with tinnitus, facial paralysis, and vesicles of the external ear.
What is the most likely diagnosis?


A. Primary Herpes Simplex infection

B. Coxsackievirus A16 infection

C. Cytomegalovirus infection

D. Primary Varicella Infection

E. Herpes Zoster InfectionCorrect Choice
This patient has Ramsay Hunt Syndrome, herpes zoster infection of the geniculate ganglion.
Vesicles can be seen on the tympanic membrane and the external ear. Symptoms include ipsilateral
facial paralysis, tinnitus, or other auditory symptoms. Treatment is with acyclovir and systemic
corticosteroids


105) Calabar swellings are a characteristic feature of which disease?

A. Eumycetoma

B. LoiasisCorrect Choice

C. Carrion's Disease

D. Tertiary Syphilis

E. Onchocerciasis
Calabar swellings are characteristic of Loiasis. They are transient, non-tender soft tissue swellings
found most commonly around joints


106) What is the characteristic body on histopathology for this disease?



                                                  27
A. Henderson-Patterson

B. Cowdry Type ACorrect Choice

C. Dutcher

D. Guarnieri

E. Cowdry Type B
Herpes simlex virus is characterized by Cowdry type A bodies on histopathology. They appear as
sharply demarcated eosinophilic structures separated by a clear halo from a basophilic rim of the
infected cell's marginated nuclear chromatin. Cowdry B bodies may be seen in polio infections


107) A 10 year-old girl presents to the dermatology clinic with a pruritic eruption on her trunk after
swimming at a beach in Florida. Of note, the lesions are limited to skin that was covered by her
bathing suit. The most likely diagnosis is:


A. Swimmer’s itch

B. Cutaneous larva migrans

C. Cercarial dermatitis

D. Seabather’s eruption Correct Choice

E. Swimming pool granuloma
Seabather’s eruption characteristically involves areas covered by swimwear, as a result of
coelenterate larvae becoming trapped underneath bathing suits. The causative organisms are larval
forms of Edwardsiella lineata (sea anemone) and Linuche unquiculata (thimble jellyfish).


108) The Jarisch-Herxeimer reaction is most closely associated with the release of which of the
following cytokines?


A. IFN-alpha

B. IL-10

C. IL-12

D. IL-4

E. TNF-alpha Correct Choice
TNF-alpha. The Jarisch-Herxheimer reaction is associated with increased plasma concentrations of
TNF-alpha after antibiotic treatment of syphilis. The Jarisch-Herxheimer reaction has also been
described following treatment of louse-borne relapsing fever, lyme disease and leptospirosis


109) The causative organism of epidemic typhus is:

A. Salmonella typhi

B. R. prowazekiiCorrect Choice

C. Francisella tularensis

D. R. typhi

E. R. akari
Epidemic typhus is caused by infection with R. prowazekii




                                                 28
110) The expanding annular erythematous patch shown developed in a 25 year-old woman after
returning from a weekend at her summer home in Long Island, New York. The vector of this
infection is:


A. Lutzomyia verrucarum

B. Ixodes dammini Correct Choice

C. Ixodes ricinus

D. Ctenocephalides felis

E. Pediculus humanus corporis
This is an example of erythema migrans, seen in Lyme disease. Ixodes dammini is the principal
vector in the northeastern U.S


111) Streptobacillus moniliformis is the causative organism of which infectious disease?

A. Glanders

B. Rat-bite fever (Haverhill fever)Correct Choice

C. Tularemia

D. Cat scratch disease

E. Scrub typhus
Rat-bite fever (Haverhill fever) is caused by Streptobacillus moniliformis


112) Which antibiotic is best to treat this condition caused by Bartonella hensalae.

A. Ceftriaxone

B. ErythromycinCorrect Choice

C. Clindamycin

D. Fluconazole

E. Trimethoprim/sulfamethoxasole
Bartonella hensalae is the causative agent of bacillary angiomatosis. The differential diagnosis of
bacillary angiomatosis may include pyogenic granulomas and Kaposi's sarcoma. The treatment of
choice is erythromycin


113) The treatment of choice of the tropical infectious disease shown is:

A. Ivermectin

B. Mebendazole

C. Permethrin

D. Pentavalent antimony Correct Choice

E. Diethylcarbamazine
The ulcerated nodule shown here is characteristic of cutaneous leishmaniasis (CL). Leishmaniasis is
treated with pentavalent antimony


114) Tick-borne Relapsing Fever is caused by:



                                                    29
A. B. duttoniiCorrect Choice

B. B. henselae

C. B. burgdorferi

D. B. recurrentis

E. B. bacilliformis
Relapsing Fever can be either Louse-borne or Tick-borne. Tick-borne Relapsing Fever is caused by
B. duttonii, while Louse-borne Relapsing Fever is caused by B. recurrentis


115) Cat scratch disease is caused by infection with:

A. Bartonella speciesCorrect Choice

B. Gram-positive bacilli

C. Spirochetes

D. Rickettsiae

E. Gram-positive cocci
Cat scratch disease is caused by infection with B. henselae - a gram-negative bacillus of the genus
Bartonella


116) Corynebacterium minuttissimum is responsible for which of the following infections?

A. Erysipeloid

B. Trichomycosis axillarisCorrect Choice

C. Majocchi's granuloma

D. Erysipelas

E. Malakoplakia
Corynebacterium minutissimum is responsible for causing trichomycosis axillaris, a superficial
bacterial overgrowth of axillary hairs characterized by circumferential yellow (most common), red,
or black granular concretions. Majocchi's granuloma is a dermatophyte infection. Erysipelas is
caused by beta-hemolytic group A streptococcus. Malakoplakia is usually caused by S. aureus, P.
aeruginosa, or E. coli. Erysipeloid is caused by Erysipelothrix rhusiopathiae. Corynebacterium is also
the causative organism in erythrasma and pitted keratolysis


117) Measles is caused by which type of virus?

A. ParamyxovirusCorrect Choice

B. Togavirus

C. Parvovirus

D. Picornavirus

E. Rhabdovirus
Measles is caused by an RNA virus known as paramyxovirus. The illness is characterized by high
fever, cough, coryza, conjunctivitis, and Koplik's spots. Viral exanthem characterized by
erythematous macules and papules beginning on the face and spreading caudally 3-4 days after
prodromal symptoms




                                                 30
118) A 27 year-old woman who is 30 weeks pregnant presents with erythema migrans. The
treatment of choice for this patient is:


A. Doxycycline

B. AmoxicillinCorrect Choice

C. Erythromycin

D. Clindamycin

E. Chloramphenicol
In the setting of pregnancy, amoxicillin is the treatment of choice for Lyme Disease since
doxycycline is contraindicated


119) The treatment of choice for Loiasis is:

A. Diethylcarbamazine (DEC)Correct Choice

B. Permethrin

C. Ivermectin

D. Malathion

E. Chloramphenicol
The recommended treatment for loiasis is Diethylcarbamazine (DEC


120) The most likely target for exfoliative toxin A in bullous impetigo is:

A. Desmoglein 1Correct Choice

B. BPAG2

C. BPAG1

D. Desmocollin

E. Desmoglein 3
Bullous impetigo is caused primarily by staphylococcus aureus phage type 71. This bacteria
produces exfoliative toxin A binds to desmoglein 1 and produces produces the blisters characteristic
of bullous impetigo


121) The infectious disease shown is most likely caused by which of the following organisms?

A. L. mexicana complex

B. L. infantum

C. L. major

D. Klebsiella pneumoniae rhinoscleromatis Correct Choice

E. L. donovani
The figure is an example of rhinoscleroma, which is caused by Klebsiella pneumoniae
rhinoscleromatis. While the diagnosis of mucocutaneous leishmaniasis (MCL) could be considered,
none of the leismania species listed as possible responses are causes of MCL


122) A 8 year-old girl presents to the pediatrician with a poorly marginated blue-red single
indurated plaque on her right cheek. What is the most likely causative organism?



                                                  31
A. Klebsiella pneumoniae

B. Staphyloccus aureus

C. Beta-hemolytic group A streptococcus

D. Haemophilus influenzaeCorrect Choice

E. Pseudomonas aeruginosa
The most likely diagnosis is H. flu cellulitis. This infection usually occurs in children and is
characterized by a single indurated plaque with poorly delineated margins. Erysipelas, an infection
caused by beta-hemolytic streptococcus group A, usually presents with well delineated margins in
adult patients


123) A 35 year-old woman who works in the animal product industry presents with a painless
hemorrhagic bulla. A diagnosis of anthrax is suspected. Gram stain of the vesicle fluid should reveal
which of the following?


A. Gram-negative cocci

B. Guarnieri bodies

C. Gram-positive bacilli Correct Choice

D. Gram-negative bacilli

E. Gram-positive cocci
Anthrax is caused by Bacillus anthracis, which is a gram-positive spore-forming rod


124) The mechanism of action of this virulence factor of Bacillus anthracis is via increasing the
activity of tumor necrosis factor alpha:


A. Lethal toxinCorrect Choice

B. All of the above answers are correct

C. Edema toxin

D. Polyglutamate acid capsule

E. None of the above answers are correct
Bacillus anthracis is responsible for causing Anthrax. Anthrax of the skin is characterized by a
painless edematous nodule which rapidly degenerates into an eschar. B. anthracis has 3 virulence
factors: a polyglutamate acid capsule which inhibits phagocytosis of the bacteria, edema toxin
(edema factor and protective factor) which results in edema secondary to induction of cAMP
pathway, and lethal toxin (lethal factor and protective factor) which leads to shock and death via
increasing TNFalpha activity


125) At what rate of speed does this parasite migrate through the skin?

A. 0.1 cm/day

B. 10 cm/day

C. 1 cm/day

D. 2 cm/dayCorrect Choice

E. 100 cm/day




                                                 32
The infection depicted is larva migrans or creeping eruption which is caused by Ancylostoma
braziliense. This nematode is typically a hookworm of cats and dogs. It migrates at a rate of 2
cm/day. Larva currens migrates at a rate of 10 cm/day


126) High doses of which of the following antiviral agents has been associated with thrombotic
thrombocytopenic purpura in immunosuppressed patients?


A. Acyclovir

B. ValcyclovirCorrect Choice

C. Cidofovir

D. Foscarnet

E. Famciclovir
Thrombotic thrombocytopenic purpura has been reported with high doses of valcyclovir in
immunosuppressed patients


127) Orf virus infection is also known as:

A. Ecthyma

B. Pseudocowpox

C. Ecthyma contagiosumCorrect Choice

D. Milker's nodule

E. Echtyma gangrenosum
Orf virus infection is also known as Ecthyma contagiosum. Ecthyma Gangrenosum is caused by
Pseudomonas aeruginosa. Ecthyma is caused by Staphylococcal or streptococcal pyoderma.
Pseudocowpox and Milker's nodule are also known as paravaccinia and caused by udders of infected
cows


128) Botryomycosis is:

A. An acute, disseminated infection involving the genitourinary system

B. A fungal infection caused by T. tonsurans

C. A bacterial infection commonly associated with Staphylococcus aureusCorrect Choice

D. A bacterial infection commonly associated with Clostridium botulinum

E. A fungal infection caused by M. canis
Botryomycosis is an uncommon, chronic, idonlent disorder characterized by nodular, crusted,
purulent lesions. Sinuses that discharge sulfur granules are present. These heal with atrophic scars.
The granules yield most commonly Staphylococcus aureus on culture, although cases caused by
Pseudomonas aeruginosa, E. coli, Proteus, Bacteroides, and Streptococcus have been reported


129) Xenopsylla cheopis is the vector responsible for which infectious disease?

A. Human Granulocytic Ehrlichiosis

B. Scrub typhus

C. Endemic typhusCorrect Choice

D. Human Monocytic Ehrlichiosis




                                                 33
E. Epidemic typhus
Xenopsylla cheopis (rat flea) is the vector of endemic typhus, which is caused by R. typhi


130) Erythema infectiosum is caused by a:

A. None of these answers are correct

B. Single stranded DNA virusCorrect Choice

C. Double stranded DNA virus

D. RNA virus

E. Herpes virus
Erythema infectiosum is caused by parvovirus B19 which is a single stranded DNA virus


131) Which of the following vectors is responsible for transmitting Chagas’ disease?

A. Reduviid bug Correct Choice

B. Sandfly

C. Black fly

D. Tstse fly

E. Mosquito
The reduviid big is the vector of American trypanosomiasis (Chagas disease). The vector of African
trypanosomiasis is the tstse fly


132) All of the following statements are true of Bacillus anthracis infection except:

A. inhalation, GI and cutaneous forms exist

B. ciprofloxacin and doxycycline are first line treatments

C. Spores remain stable for decades

D. 20% mortality rate in untreated cutaneous infectionsCorrect Choice

E. no recommendations from CDC exist on anthrax vaccine
Anthrax can present in three different clinical forms: cutaneous, inhalational, and gastrointestinal.
The cutaneous form appears as a painless vesicle that later forms an eschar (malignant pustule).
The cutaneous form has a mortality rate of less than 1% if treated an up to 20% if untreated


133) Which of the following is the vector responsible for the transmission of Verruga Peruana?

A. Ctenocephalides felis

B. Tsetse fly

C. Lutzomyia verrucarumCorrect Choice

D. Pediculus humanus corporis

E. Simulium slossonae
Verruga Peruana is caused by infection with B. bacilliformis, which is transmitted by the sandfly,
Lutzomyia verrucarum




                                                  34
35

Contenu connexe

Tendances

MCQ August with answers - Dr Ameen Alawadhi
MCQ August with answers - Dr Ameen AlawadhiMCQ August with answers - Dr Ameen Alawadhi
MCQ August with answers - Dr Ameen Alawadhiaskadermatologist
 
ETAS_MCQ_10 manifestations of systemic diseases1
ETAS_MCQ_10 manifestations of systemic diseases1ETAS_MCQ_10 manifestations of systemic diseases1
ETAS_MCQ_10 manifestations of systemic diseases1Derma202
 
Dermatology MCQ and AAFP.pptx
Dermatology MCQ and AAFP.pptxDermatology MCQ and AAFP.pptx
Dermatology MCQ and AAFP.pptxAbdulaziz Bagasi
 
MCQ July with answers - Dr Ameen Alawadhi
MCQ July with answers - Dr Ameen AlawadhiMCQ July with answers - Dr Ameen Alawadhi
MCQ July with answers - Dr Ameen Alawadhiaskadermatologist
 
ETAS_MCQ_15 dermatologic and cosmetic surgery
ETAS_MCQ_15 dermatologic and cosmetic surgeryETAS_MCQ_15 dermatologic and cosmetic surgery
ETAS_MCQ_15 dermatologic and cosmetic surgeryDerma202
 
Dermatology board review
Dermatology board reviewDermatology board review
Dermatology board reviewAhmed Amer
 
OSCE 1 and 2 combined - Dr Ameen Alawadhi
OSCE 1 and 2 combined - Dr Ameen AlawadhiOSCE 1 and 2 combined - Dr Ameen Alawadhi
OSCE 1 and 2 combined - Dr Ameen Alawadhiaskadermatologist
 
ETAS_MCQ_05 dermatopathology
ETAS_MCQ_05 dermatopathologyETAS_MCQ_05 dermatopathology
ETAS_MCQ_05 dermatopathologyDerma202
 
Parasitology revision 2016 mod ug
Parasitology revision 2016 mod ugParasitology revision 2016 mod ug
Parasitology revision 2016 mod ugMohammed Sarhan
 
Mcqs infectious diseases 08
Mcqs infectious diseases      08Mcqs infectious diseases      08
Mcqs infectious diseases 08DOCTOR WHO
 
MCQs for UG students
MCQs for UG  studentsMCQs for UG  students
MCQs for UG studentsSuprakash Das
 
dermatology.Disorders of keratinization.(dr.darseem)
dermatology.Disorders of keratinization.(dr.darseem)dermatology.Disorders of keratinization.(dr.darseem)
dermatology.Disorders of keratinization.(dr.darseem)student
 
Haemophilus species
Haemophilus speciesHaemophilus species
Haemophilus speciesBruno Mmassy
 
Previous year question on pemphigus vulgaris based on neet pg, usmle, plab an...
Previous year question on pemphigus vulgaris based on neet pg, usmle, plab an...Previous year question on pemphigus vulgaris based on neet pg, usmle, plab an...
Previous year question on pemphigus vulgaris based on neet pg, usmle, plab an...Medico Apps
 
Mock question paper for neet pg, usmle, plab and fmge (mci screening exam) on...
Mock question paper for neet pg, usmle, plab and fmge (mci screening exam) on...Mock question paper for neet pg, usmle, plab and fmge (mci screening exam) on...
Mock question paper for neet pg, usmle, plab and fmge (mci screening exam) on...Medico Apps
 
Bullous diseases(group a)
Bullous diseases(group a)Bullous diseases(group a)
Bullous diseases(group a)Habrol Afzam
 

Tendances (20)

MCQ August with answers - Dr Ameen Alawadhi
MCQ August with answers - Dr Ameen AlawadhiMCQ August with answers - Dr Ameen Alawadhi
MCQ August with answers - Dr Ameen Alawadhi
 
ETAS_MCQ_10 manifestations of systemic diseases1
ETAS_MCQ_10 manifestations of systemic diseases1ETAS_MCQ_10 manifestations of systemic diseases1
ETAS_MCQ_10 manifestations of systemic diseases1
 
Dermatology MCQ and AAFP.pptx
Dermatology MCQ and AAFP.pptxDermatology MCQ and AAFP.pptx
Dermatology MCQ and AAFP.pptx
 
MCQ July with answers - Dr Ameen Alawadhi
MCQ July with answers - Dr Ameen AlawadhiMCQ July with answers - Dr Ameen Alawadhi
MCQ July with answers - Dr Ameen Alawadhi
 
ETAS_MCQ_15 dermatologic and cosmetic surgery
ETAS_MCQ_15 dermatologic and cosmetic surgeryETAS_MCQ_15 dermatologic and cosmetic surgery
ETAS_MCQ_15 dermatologic and cosmetic surgery
 
Dermatology board review
Dermatology board reviewDermatology board review
Dermatology board review
 
OSCE 1 and 2 combined - Dr Ameen Alawadhi
OSCE 1 and 2 combined - Dr Ameen AlawadhiOSCE 1 and 2 combined - Dr Ameen Alawadhi
OSCE 1 and 2 combined - Dr Ameen Alawadhi
 
ETAS_MCQ_05 dermatopathology
ETAS_MCQ_05 dermatopathologyETAS_MCQ_05 dermatopathology
ETAS_MCQ_05 dermatopathology
 
Parasitology revision 2016 mod ug
Parasitology revision 2016 mod ugParasitology revision 2016 mod ug
Parasitology revision 2016 mod ug
 
Mcqs infectious diseases 08
Mcqs infectious diseases      08Mcqs infectious diseases      08
Mcqs infectious diseases 08
 
MCQs for UG students
MCQs for UG  studentsMCQs for UG  students
MCQs for UG students
 
dermatology.Disorders of keratinization.(dr.darseem)
dermatology.Disorders of keratinization.(dr.darseem)dermatology.Disorders of keratinization.(dr.darseem)
dermatology.Disorders of keratinization.(dr.darseem)
 
Haemophilus species
Haemophilus speciesHaemophilus species
Haemophilus species
 
Ichthyosis
IchthyosisIchthyosis
Ichthyosis
 
Leprosy
LeprosyLeprosy
Leprosy
 
Patch Testing: A to Z
Patch Testing: A to ZPatch Testing: A to Z
Patch Testing: A to Z
 
Chromomycosis
ChromomycosisChromomycosis
Chromomycosis
 
Previous year question on pemphigus vulgaris based on neet pg, usmle, plab an...
Previous year question on pemphigus vulgaris based on neet pg, usmle, plab an...Previous year question on pemphigus vulgaris based on neet pg, usmle, plab an...
Previous year question on pemphigus vulgaris based on neet pg, usmle, plab an...
 
Mock question paper for neet pg, usmle, plab and fmge (mci screening exam) on...
Mock question paper for neet pg, usmle, plab and fmge (mci screening exam) on...Mock question paper for neet pg, usmle, plab and fmge (mci screening exam) on...
Mock question paper for neet pg, usmle, plab and fmge (mci screening exam) on...
 
Bullous diseases(group a)
Bullous diseases(group a)Bullous diseases(group a)
Bullous diseases(group a)
 

En vedette

ETAS_MCQ_11 disorder of hair and nails
ETAS_MCQ_11 disorder of hair and nailsETAS_MCQ_11 disorder of hair and nails
ETAS_MCQ_11 disorder of hair and nailsDerma202
 
ETAS_MCQ_03 a genodermatoses
ETAS_MCQ_03 a genodermatosesETAS_MCQ_03 a genodermatoses
ETAS_MCQ_03 a genodermatosesDerma202
 
Phototherapy treatment protocol
Phototherapy treatment protocolPhototherapy treatment protocol
Phototherapy treatment protocolDerma202
 
MCQ test item analysis
MCQ test item analysisMCQ test item analysis
MCQ test item analysisSoha Rashed
 
Spring Cleaning: 7 things to give up to grow your business
Spring Cleaning: 7 things to give up to grow your businessSpring Cleaning: 7 things to give up to grow your business
Spring Cleaning: 7 things to give up to grow your businessDave Kerpen
 

En vedette (7)

ETAS_MCQ_11 disorder of hair and nails
ETAS_MCQ_11 disorder of hair and nailsETAS_MCQ_11 disorder of hair and nails
ETAS_MCQ_11 disorder of hair and nails
 
ETAS_MCQ_03 a genodermatoses
ETAS_MCQ_03 a genodermatosesETAS_MCQ_03 a genodermatoses
ETAS_MCQ_03 a genodermatoses
 
Phototherapy treatment protocol
Phototherapy treatment protocolPhototherapy treatment protocol
Phototherapy treatment protocol
 
Mrcp 2 dermatology
Mrcp 2 dermatologyMrcp 2 dermatology
Mrcp 2 dermatology
 
Dermatology made easy
Dermatology made easyDermatology made easy
Dermatology made easy
 
MCQ test item analysis
MCQ test item analysisMCQ test item analysis
MCQ test item analysis
 
Spring Cleaning: 7 things to give up to grow your business
Spring Cleaning: 7 things to give up to grow your businessSpring Cleaning: 7 things to give up to grow your business
Spring Cleaning: 7 things to give up to grow your business
 

Similaire à ETAS_MCQ_08 infectious diseases of the skin

Similaire à ETAS_MCQ_08 infectious diseases of the skin (20)

Measles made Ridiculously Easy!!!!!!!!!!
Measles made Ridiculously Easy!!!!!!!!!!Measles made Ridiculously Easy!!!!!!!!!!
Measles made Ridiculously Easy!!!!!!!!!!
 
Dna virus
Dna virusDna virus
Dna virus
 
Systemic_MycosesM
Systemic_MycosesMSystemic_MycosesM
Systemic_MycosesM
 
National Dengue Management Guideline for Bangladesh
National Dengue Management Guideline for BangladeshNational Dengue Management Guideline for Bangladesh
National Dengue Management Guideline for Bangladesh
 
Coccidian parasite
Coccidian parasiteCoccidian parasite
Coccidian parasite
 
Aids
AidsAids
Aids
 
Crimson Publishers-Overview of Cutaneous Tuberculosis
Crimson Publishers-Overview of Cutaneous TuberculosisCrimson Publishers-Overview of Cutaneous Tuberculosis
Crimson Publishers-Overview of Cutaneous Tuberculosis
 
Viral Diseases
Viral DiseasesViral Diseases
Viral Diseases
 
MALARIA
MALARIAMALARIA
MALARIA
 
Smallpox disease
Smallpox diseaseSmallpox disease
Smallpox disease
 
Human health & diseases L03
Human health & diseases L03Human health & diseases L03
Human health & diseases L03
 
Ganyang MCQ Infectious dss
Ganyang MCQ Infectious dssGanyang MCQ Infectious dss
Ganyang MCQ Infectious dss
 
Infectious diseases of camelids
Infectious diseases of camelidsInfectious diseases of camelids
Infectious diseases of camelids
 
Skin
SkinSkin
Skin
 
bioterrorism-170215073558.pdf
bioterrorism-170215073558.pdfbioterrorism-170215073558.pdf
bioterrorism-170215073558.pdf
 
Bioterrorism
BioterrorismBioterrorism
Bioterrorism
 
Infection
InfectionInfection
Infection
 
Monkeypox.pdf
Monkeypox.pdfMonkeypox.pdf
Monkeypox.pdf
 
Poxvirus
PoxvirusPoxvirus
Poxvirus
 
Emerging pathogens
Emerging pathogensEmerging pathogens
Emerging pathogens
 

Plus de Derma202

Histopathplogical photos
Histopathplogical photosHistopathplogical photos
Histopathplogical photosDerma202
 
Slide study from ETAS
Slide  study from ETASSlide  study from ETAS
Slide study from ETASDerma202
 
Dermatology
DermatologyDermatology
DermatologyDerma202
 
ETAS_MCQ_14 plants and creatures of dermatologic significance
ETAS_MCQ_14 plants and creatures of dermatologic significanceETAS_MCQ_14 plants and creatures of dermatologic significance
ETAS_MCQ_14 plants and creatures of dermatologic significanceDerma202
 
ETAS_MCQ_13 photobiology and photosensitivity disorders
ETAS_MCQ_13 photobiology and photosensitivity disordersETAS_MCQ_13 photobiology and photosensitivity disorders
ETAS_MCQ_13 photobiology and photosensitivity disordersDerma202
 
ETAS_MCQ_03 b genodermatoses
ETAS_MCQ_03 b genodermatosesETAS_MCQ_03 b genodermatoses
ETAS_MCQ_03 b genodermatosesDerma202
 
Derm handbook for medical students and junior doctors 2010
Derm handbook for medical students and junior doctors 2010Derm handbook for medical students and junior doctors 2010
Derm handbook for medical students and junior doctors 2010Derma202
 

Plus de Derma202 (7)

Histopathplogical photos
Histopathplogical photosHistopathplogical photos
Histopathplogical photos
 
Slide study from ETAS
Slide  study from ETASSlide  study from ETAS
Slide study from ETAS
 
Dermatology
DermatologyDermatology
Dermatology
 
ETAS_MCQ_14 plants and creatures of dermatologic significance
ETAS_MCQ_14 plants and creatures of dermatologic significanceETAS_MCQ_14 plants and creatures of dermatologic significance
ETAS_MCQ_14 plants and creatures of dermatologic significance
 
ETAS_MCQ_13 photobiology and photosensitivity disorders
ETAS_MCQ_13 photobiology and photosensitivity disordersETAS_MCQ_13 photobiology and photosensitivity disorders
ETAS_MCQ_13 photobiology and photosensitivity disorders
 
ETAS_MCQ_03 b genodermatoses
ETAS_MCQ_03 b genodermatosesETAS_MCQ_03 b genodermatoses
ETAS_MCQ_03 b genodermatoses
 
Derm handbook for medical students and junior doctors 2010
Derm handbook for medical students and junior doctors 2010Derm handbook for medical students and junior doctors 2010
Derm handbook for medical students and junior doctors 2010
 

Dernier

Shazia Iqbal 2024 - Bioorganic Chemistry.pdf
Shazia Iqbal 2024 - Bioorganic Chemistry.pdfShazia Iqbal 2024 - Bioorganic Chemistry.pdf
Shazia Iqbal 2024 - Bioorganic Chemistry.pdfTrustlife
 
Jaipur Call Girl Service 📞9xx000xx09📞Just Call Divya📲 Call Girl In Jaipur No💰...
Jaipur Call Girl Service 📞9xx000xx09📞Just Call Divya📲 Call Girl In Jaipur No💰...Jaipur Call Girl Service 📞9xx000xx09📞Just Call Divya📲 Call Girl In Jaipur No💰...
Jaipur Call Girl Service 📞9xx000xx09📞Just Call Divya📲 Call Girl In Jaipur No💰...Sheetaleventcompany
 
Call 8250092165 Patna Call Girls ₹4.5k Cash Payment With Room Delivery
Call 8250092165 Patna Call Girls ₹4.5k Cash Payment With Room DeliveryCall 8250092165 Patna Call Girls ₹4.5k Cash Payment With Room Delivery
Call 8250092165 Patna Call Girls ₹4.5k Cash Payment With Room DeliveryJyoti singh
 
Gastric Cancer: Сlinical Implementation of Artificial Intelligence, Synergeti...
Gastric Cancer: Сlinical Implementation of Artificial Intelligence, Synergeti...Gastric Cancer: Сlinical Implementation of Artificial Intelligence, Synergeti...
Gastric Cancer: Сlinical Implementation of Artificial Intelligence, Synergeti...Oleg Kshivets
 
Jual Obat Aborsi Di Dubai UAE Wa 0838-4800-7379 Obat Penggugur Kandungan Cytotec
Jual Obat Aborsi Di Dubai UAE Wa 0838-4800-7379 Obat Penggugur Kandungan CytotecJual Obat Aborsi Di Dubai UAE Wa 0838-4800-7379 Obat Penggugur Kandungan Cytotec
Jual Obat Aborsi Di Dubai UAE Wa 0838-4800-7379 Obat Penggugur Kandungan Cytotecjualobat34
 
💚Chandigarh Call Girls 💯Riya 📲🔝8868886958🔝Call Girls In Chandigarh No💰Advance...
💚Chandigarh Call Girls 💯Riya 📲🔝8868886958🔝Call Girls In Chandigarh No💰Advance...💚Chandigarh Call Girls 💯Riya 📲🔝8868886958🔝Call Girls In Chandigarh No💰Advance...
💚Chandigarh Call Girls 💯Riya 📲🔝8868886958🔝Call Girls In Chandigarh No💰Advance...Sheetaleventcompany
 
Pune Call Girl Service 📞9xx000xx09📞Just Call Divya📲 Call Girl In Pune No💰Adva...
Pune Call Girl Service 📞9xx000xx09📞Just Call Divya📲 Call Girl In Pune No💰Adva...Pune Call Girl Service 📞9xx000xx09📞Just Call Divya📲 Call Girl In Pune No💰Adva...
Pune Call Girl Service 📞9xx000xx09📞Just Call Divya📲 Call Girl In Pune No💰Adva...Sheetaleventcompany
 
👉 Amritsar Call Girls 👉📞 8725944379 👉📞 Just📲 Call Ruhi Call Girl Near Me Amri...
👉 Amritsar Call Girls 👉📞 8725944379 👉📞 Just📲 Call Ruhi Call Girl Near Me Amri...👉 Amritsar Call Girls 👉📞 8725944379 👉📞 Just📲 Call Ruhi Call Girl Near Me Amri...
👉 Amritsar Call Girls 👉📞 8725944379 👉📞 Just📲 Call Ruhi Call Girl Near Me Amri...Sheetaleventcompany
 
Dehradun Call Girl Service ❤️🍑 8854095900 👄🫦Independent Escort Service Dehradun
Dehradun Call Girl Service ❤️🍑 8854095900 👄🫦Independent Escort Service DehradunDehradun Call Girl Service ❤️🍑 8854095900 👄🫦Independent Escort Service Dehradun
Dehradun Call Girl Service ❤️🍑 8854095900 👄🫦Independent Escort Service DehradunSheetaleventcompany
 
Chandigarh Call Girls Service ❤️🍑 9809698092 👄🫦Independent Escort Service Cha...
Chandigarh Call Girls Service ❤️🍑 9809698092 👄🫦Independent Escort Service Cha...Chandigarh Call Girls Service ❤️🍑 9809698092 👄🫦Independent Escort Service Cha...
Chandigarh Call Girls Service ❤️🍑 9809698092 👄🫦Independent Escort Service Cha...Sheetaleventcompany
 
Intramuscular & Intravenous Injection.pptx
Intramuscular & Intravenous Injection.pptxIntramuscular & Intravenous Injection.pptx
Intramuscular & Intravenous Injection.pptxsaranpratha12
 
💚Chandigarh Call Girls Service 💯Piya 📲🔝8868886958🔝Call Girls In Chandigarh No...
💚Chandigarh Call Girls Service 💯Piya 📲🔝8868886958🔝Call Girls In Chandigarh No...💚Chandigarh Call Girls Service 💯Piya 📲🔝8868886958🔝Call Girls In Chandigarh No...
💚Chandigarh Call Girls Service 💯Piya 📲🔝8868886958🔝Call Girls In Chandigarh No...Sheetaleventcompany
 
Genuine Call Girls Hyderabad 9630942363 Book High Profile Call Girl in Hydera...
Genuine Call Girls Hyderabad 9630942363 Book High Profile Call Girl in Hydera...Genuine Call Girls Hyderabad 9630942363 Book High Profile Call Girl in Hydera...
Genuine Call Girls Hyderabad 9630942363 Book High Profile Call Girl in Hydera...GENUINE ESCORT AGENCY
 
Cardiac Output, Venous Return, and Their Regulation
Cardiac Output, Venous Return, and Their RegulationCardiac Output, Venous Return, and Their Regulation
Cardiac Output, Venous Return, and Their RegulationMedicoseAcademics
 
Premium Call Girls Dehradun {8854095900} ❤️VVIP ANJU Call Girls in Dehradun U...
Premium Call Girls Dehradun {8854095900} ❤️VVIP ANJU Call Girls in Dehradun U...Premium Call Girls Dehradun {8854095900} ❤️VVIP ANJU Call Girls in Dehradun U...
Premium Call Girls Dehradun {8854095900} ❤️VVIP ANJU Call Girls in Dehradun U...Sheetaleventcompany
 
Whitefield { Call Girl in Bangalore ₹7.5k Pick Up & Drop With Cash Payment 63...
Whitefield { Call Girl in Bangalore ₹7.5k Pick Up & Drop With Cash Payment 63...Whitefield { Call Girl in Bangalore ₹7.5k Pick Up & Drop With Cash Payment 63...
Whitefield { Call Girl in Bangalore ₹7.5k Pick Up & Drop With Cash Payment 63...dishamehta3332
 
❤️Chandigarh Escorts Service☎️9814379184☎️ Call Girl service in Chandigarh☎️ ...
❤️Chandigarh Escorts Service☎️9814379184☎️ Call Girl service in Chandigarh☎️ ...❤️Chandigarh Escorts Service☎️9814379184☎️ Call Girl service in Chandigarh☎️ ...
❤️Chandigarh Escorts Service☎️9814379184☎️ Call Girl service in Chandigarh☎️ ...Sheetaleventcompany
 
tongue disease lecture Dr Assadawy legacy
tongue disease lecture Dr Assadawy legacytongue disease lecture Dr Assadawy legacy
tongue disease lecture Dr Assadawy legacyDrMohamed Assadawy
 
Kolkata Call Girls Shobhabazar 💯Call Us 🔝 8005736733 🔝 💃 Top Class Call Gir...
Kolkata Call Girls Shobhabazar  💯Call Us 🔝 8005736733 🔝 💃  Top Class Call Gir...Kolkata Call Girls Shobhabazar  💯Call Us 🔝 8005736733 🔝 💃  Top Class Call Gir...
Kolkata Call Girls Shobhabazar 💯Call Us 🔝 8005736733 🔝 💃 Top Class Call Gir...Namrata Singh
 
7 steps How to prevent Thalassemia : Dr Sharda Jain & Vandana Gupta
7 steps How to prevent Thalassemia : Dr Sharda Jain & Vandana Gupta7 steps How to prevent Thalassemia : Dr Sharda Jain & Vandana Gupta
7 steps How to prevent Thalassemia : Dr Sharda Jain & Vandana GuptaLifecare Centre
 

Dernier (20)

Shazia Iqbal 2024 - Bioorganic Chemistry.pdf
Shazia Iqbal 2024 - Bioorganic Chemistry.pdfShazia Iqbal 2024 - Bioorganic Chemistry.pdf
Shazia Iqbal 2024 - Bioorganic Chemistry.pdf
 
Jaipur Call Girl Service 📞9xx000xx09📞Just Call Divya📲 Call Girl In Jaipur No💰...
Jaipur Call Girl Service 📞9xx000xx09📞Just Call Divya📲 Call Girl In Jaipur No💰...Jaipur Call Girl Service 📞9xx000xx09📞Just Call Divya📲 Call Girl In Jaipur No💰...
Jaipur Call Girl Service 📞9xx000xx09📞Just Call Divya📲 Call Girl In Jaipur No💰...
 
Call 8250092165 Patna Call Girls ₹4.5k Cash Payment With Room Delivery
Call 8250092165 Patna Call Girls ₹4.5k Cash Payment With Room DeliveryCall 8250092165 Patna Call Girls ₹4.5k Cash Payment With Room Delivery
Call 8250092165 Patna Call Girls ₹4.5k Cash Payment With Room Delivery
 
Gastric Cancer: Сlinical Implementation of Artificial Intelligence, Synergeti...
Gastric Cancer: Сlinical Implementation of Artificial Intelligence, Synergeti...Gastric Cancer: Сlinical Implementation of Artificial Intelligence, Synergeti...
Gastric Cancer: Сlinical Implementation of Artificial Intelligence, Synergeti...
 
Jual Obat Aborsi Di Dubai UAE Wa 0838-4800-7379 Obat Penggugur Kandungan Cytotec
Jual Obat Aborsi Di Dubai UAE Wa 0838-4800-7379 Obat Penggugur Kandungan CytotecJual Obat Aborsi Di Dubai UAE Wa 0838-4800-7379 Obat Penggugur Kandungan Cytotec
Jual Obat Aborsi Di Dubai UAE Wa 0838-4800-7379 Obat Penggugur Kandungan Cytotec
 
💚Chandigarh Call Girls 💯Riya 📲🔝8868886958🔝Call Girls In Chandigarh No💰Advance...
💚Chandigarh Call Girls 💯Riya 📲🔝8868886958🔝Call Girls In Chandigarh No💰Advance...💚Chandigarh Call Girls 💯Riya 📲🔝8868886958🔝Call Girls In Chandigarh No💰Advance...
💚Chandigarh Call Girls 💯Riya 📲🔝8868886958🔝Call Girls In Chandigarh No💰Advance...
 
Pune Call Girl Service 📞9xx000xx09📞Just Call Divya📲 Call Girl In Pune No💰Adva...
Pune Call Girl Service 📞9xx000xx09📞Just Call Divya📲 Call Girl In Pune No💰Adva...Pune Call Girl Service 📞9xx000xx09📞Just Call Divya📲 Call Girl In Pune No💰Adva...
Pune Call Girl Service 📞9xx000xx09📞Just Call Divya📲 Call Girl In Pune No💰Adva...
 
👉 Amritsar Call Girls 👉📞 8725944379 👉📞 Just📲 Call Ruhi Call Girl Near Me Amri...
👉 Amritsar Call Girls 👉📞 8725944379 👉📞 Just📲 Call Ruhi Call Girl Near Me Amri...👉 Amritsar Call Girls 👉📞 8725944379 👉📞 Just📲 Call Ruhi Call Girl Near Me Amri...
👉 Amritsar Call Girls 👉📞 8725944379 👉📞 Just📲 Call Ruhi Call Girl Near Me Amri...
 
Dehradun Call Girl Service ❤️🍑 8854095900 👄🫦Independent Escort Service Dehradun
Dehradun Call Girl Service ❤️🍑 8854095900 👄🫦Independent Escort Service DehradunDehradun Call Girl Service ❤️🍑 8854095900 👄🫦Independent Escort Service Dehradun
Dehradun Call Girl Service ❤️🍑 8854095900 👄🫦Independent Escort Service Dehradun
 
Chandigarh Call Girls Service ❤️🍑 9809698092 👄🫦Independent Escort Service Cha...
Chandigarh Call Girls Service ❤️🍑 9809698092 👄🫦Independent Escort Service Cha...Chandigarh Call Girls Service ❤️🍑 9809698092 👄🫦Independent Escort Service Cha...
Chandigarh Call Girls Service ❤️🍑 9809698092 👄🫦Independent Escort Service Cha...
 
Intramuscular & Intravenous Injection.pptx
Intramuscular & Intravenous Injection.pptxIntramuscular & Intravenous Injection.pptx
Intramuscular & Intravenous Injection.pptx
 
💚Chandigarh Call Girls Service 💯Piya 📲🔝8868886958🔝Call Girls In Chandigarh No...
💚Chandigarh Call Girls Service 💯Piya 📲🔝8868886958🔝Call Girls In Chandigarh No...💚Chandigarh Call Girls Service 💯Piya 📲🔝8868886958🔝Call Girls In Chandigarh No...
💚Chandigarh Call Girls Service 💯Piya 📲🔝8868886958🔝Call Girls In Chandigarh No...
 
Genuine Call Girls Hyderabad 9630942363 Book High Profile Call Girl in Hydera...
Genuine Call Girls Hyderabad 9630942363 Book High Profile Call Girl in Hydera...Genuine Call Girls Hyderabad 9630942363 Book High Profile Call Girl in Hydera...
Genuine Call Girls Hyderabad 9630942363 Book High Profile Call Girl in Hydera...
 
Cardiac Output, Venous Return, and Their Regulation
Cardiac Output, Venous Return, and Their RegulationCardiac Output, Venous Return, and Their Regulation
Cardiac Output, Venous Return, and Their Regulation
 
Premium Call Girls Dehradun {8854095900} ❤️VVIP ANJU Call Girls in Dehradun U...
Premium Call Girls Dehradun {8854095900} ❤️VVIP ANJU Call Girls in Dehradun U...Premium Call Girls Dehradun {8854095900} ❤️VVIP ANJU Call Girls in Dehradun U...
Premium Call Girls Dehradun {8854095900} ❤️VVIP ANJU Call Girls in Dehradun U...
 
Whitefield { Call Girl in Bangalore ₹7.5k Pick Up & Drop With Cash Payment 63...
Whitefield { Call Girl in Bangalore ₹7.5k Pick Up & Drop With Cash Payment 63...Whitefield { Call Girl in Bangalore ₹7.5k Pick Up & Drop With Cash Payment 63...
Whitefield { Call Girl in Bangalore ₹7.5k Pick Up & Drop With Cash Payment 63...
 
❤️Chandigarh Escorts Service☎️9814379184☎️ Call Girl service in Chandigarh☎️ ...
❤️Chandigarh Escorts Service☎️9814379184☎️ Call Girl service in Chandigarh☎️ ...❤️Chandigarh Escorts Service☎️9814379184☎️ Call Girl service in Chandigarh☎️ ...
❤️Chandigarh Escorts Service☎️9814379184☎️ Call Girl service in Chandigarh☎️ ...
 
tongue disease lecture Dr Assadawy legacy
tongue disease lecture Dr Assadawy legacytongue disease lecture Dr Assadawy legacy
tongue disease lecture Dr Assadawy legacy
 
Kolkata Call Girls Shobhabazar 💯Call Us 🔝 8005736733 🔝 💃 Top Class Call Gir...
Kolkata Call Girls Shobhabazar  💯Call Us 🔝 8005736733 🔝 💃  Top Class Call Gir...Kolkata Call Girls Shobhabazar  💯Call Us 🔝 8005736733 🔝 💃  Top Class Call Gir...
Kolkata Call Girls Shobhabazar 💯Call Us 🔝 8005736733 🔝 💃 Top Class Call Gir...
 
7 steps How to prevent Thalassemia : Dr Sharda Jain & Vandana Gupta
7 steps How to prevent Thalassemia : Dr Sharda Jain & Vandana Gupta7 steps How to prevent Thalassemia : Dr Sharda Jain & Vandana Gupta
7 steps How to prevent Thalassemia : Dr Sharda Jain & Vandana Gupta
 

ETAS_MCQ_08 infectious diseases of the skin

  • 1. Infectious diseases of the skin 1) The causative organism for Brucellosis is which of the following: A. Acid fast bacilli B. Gram positive cocci C. Gram negative rodCorrect Choice D. Gram positive rod E. Gram negative cocci Brucella species which cause Brucellosis are gram negative rods. Brucellosis, aka undulant fever, is characterized by an acute febrile illness with headache and joint pain. CNS and cardiac manifestations can also occur. It is acquired by contact with infected animals or contaminated dairy products. Treatment is with doxycycline and rifampin 2) What is the most likely diagnosis of this lesion occurring on the hand of a butcher? A. Heck's disease B. Butcher's wart C. Contagious ecthymaCorrect Choice D. Molloscum contagiosum E. Ecthyma gangrenosum Contagious ecthyma, or orf, occurs most frequently among shepherds and butchers as it is a viral disease that is more common among sheeps and goats. It is caused by a farmyard pox, which also cause Milker's nodules. The lesions tend to heal spontaneously after several weeks 3) For which of the following smallpox vaccination complications is the administration of vaccine immune globulin indicated? A. Vaccinia keratitis B. Mild generalized vaccinia C. Post-vaccinal encephalitis D. Eczema vaccinatumCorrect Choice E. Erythema multiforme Vaccinia immune globulin can be administered to treat some of the complications associated with vaccinia vaccination. Vaccinia immune globulin is indicated for inadvertent inoculation with extensive involvement or ocular implantations, eczema vaccinatum, generalized vaccinia (severe or recurrent), and progressive vaccinia. It is not recommended for inadvertent inoculation with mild reaction, generalized vaccinia (mild or limited), post-vaccination encephalitis, and isolated vaccinia keratitis 4) What characteristic color is seen on diascopy of this lesion? A. Coral red B. Orange C. Blue 1
  • 2. D. Red E. Apple jellyCorrect Choice Leishmaniasis recidivans is a type of Old World leishmaniasis. Clinically, it appears as a red papule covered with white scale. On diascopy, it has a characteristic apple jelly color 5) Measles is caused by a: A. Togavirus B. Rhabdovirus C. Picornavirus D. ParamyxovirusCorrect Choice E. Adenovirus The measles virus is an RNA virus belonging to the Paramyxovirus family 6) Contagious pustular dermatitis (Orf) is caused by a: A. Poxvirus Correct Choice B. Gram-positive spore-forming rod C. Papovavirus D. Herpesvirus E. Paramyxovirus Orf is caused by orf virus (OV), a poxvirus of the genus Parapoxvirus 7) Butcher's warts are caused by which human papilloma virus (HPV) type? A. 7Correct Choice B. 2 C. 5 D. 3 E. 13 Butcher's warts are caused by HPV type 7 8) An 18-year old man presents to the Dermatology Clinic with a nontender penile erosion that has been present for 2 weeks. An indurated border and nontender bilateral inguinal lymphadenopathy are also noted. What is the most likely diagnosis? A. Lymphogranuloma venereum B. Herpes simplex C. Chancroid D. Primary syphilisCorrect Choice E. HIV A painless chancre with an indurated border is characteristic of primary syphilis. Associated painless lymphadenopathy ("buboes") is also a common feature 2
  • 3. 9) What is the causative agent of this parasitic eruption which also causes patchy pulmonary infiltrate and eosinophilia? A. Filariasis B. Dracunculiasis C. Larva migransCorrect Choice D. Larva currens E. Gnathostomiasis Larva migrans is caused by Ancylostoma braziliense, a hookworm of cats and dogs. It also associated with Loeffler's syndrome, patchy infiltrate of the lungs with eosinophilia. Treatment for this parasitic infection is ivermectin or albendazole 10) One of your colleagues returned from a vacation to India with fever, vomiting, pleural effusions, ascites and conjunctival petechiae. She also has a diffuse macular rash with notable areas of sparing that your attending refers to as “white islands in a sea of red”. What is your diagnosis? A. Malaria B. Leishmania C. Leptospirosis D. Typhoid E. Dengue hemorrhagic feverCorrect Choice Dengue fever is caused by an Arbovirus and transmitted by the mosquito, Aedes aegypti and may cause Dengue Shock Syndrome and Dengue Hemorrhagic Fever. The infection starts with sudden onset of high fever, backache, retroorbital pain, bone and joint pain, depression and malaise. The disease is also called "break-bone fever." One to seven days after onset of fever, rash presents characteristically starting on the dorsum and hands and feet spreading to limbs and torso. The eruption may become confluent with small, round islands of sparing, the so called "white islands in a sea of red." Treatment is generally supportive as no antivirals are effective 11) The treatment of choice for acyclovir-resistant herpes simplex virus infection is: A. Saquinivir B. Valcyclovir C. Indinivir D. FoscarnetCorrect Choice E. Famciclovir Foscarnet directly inhibits viral DNA polymerase (without requiring phosphorylation by TK) and is therefore effective in acyclovir-resistant HSV infections. Cidofovir can also be used 12) Tricomycosis axillaris is caused by: A. Burkholderia mallei B. Corynebacterium tenuis Correct Choice C. Proteus species 3
  • 4. D. Micrococcus sedentarius E. Corynebacterium minitissimum Corynebacterium tenuis 13) Which of the following are inconsistent with the diagnosis of staphylococcal scalded skin syndrome? A. Prognosis is good in children with low mortality when anitbiotics are administered early B. Swabs and cultures of fluid filled blisters overwhelmingly grow staphCorrect Choice C. Initial event is usually a localized staph infection D. Cell necrosis does not occur in staphylococcal scalded skin syndrome as it does in T.E.N E. Epidermal changes are produced by exfoliative toxins of staphlococcus Staphylococcal scalded skin syndrome: Lesions extend far beyond areas of actual staphylococcal infection, by action of the epidermolytic exotoxin elaborated by the staphylococcus in remote sites. Usually the staphylococci are present at a distant focus such as the parynx, nose ear, or conjuctiva. If cultures are taken they should be obtained fromthe mucous membranes because the skin erythema and desquamation is due to the distant effects of the exfoliative toxin, unlike the sitaution in bullous impetigo, where S. aureus is present in the lesions. Epidermal changes are produced by exfoliative toxins of Staphylococcus. Inital event is usually a localized Staph infection. Prognosis is good in children with low mortality when anitbiotics are administered earyl. Cell necrosis does not occur in SSS as it does in toxic epidermal necrolysis 14) Which of the following is the vector for Dengue fever? A. Lutzomyia verrucarum B. Phlebotomus papatasii C. Xenopsylla cheopis D. Aedes aegyptiCorrect Choice E. Culex mosquito Dengue fever also known as "break-bone" fever is characterized by sudden high fever, backache, retro-orbital pain, bone/joint pain, weakness, and malaise. It is caused by an arbovirus (RNA virus) which is transmitted by Aedes aegypti, a species of mosquito 15) Which virus is most commonly associated with oral hairy leukoplakia? A. Human papilloma virus B. Epstein-Barr virusCorrect Choice C. Parvovirus D. Pox virus E. Herpes virus Oral hairy leukoplakia is an oral mucosal infection most often caused by the Epstein-Barr virus, which occurs in immunocompromised patients. Clinically, thick, white plaques are noted on the lateral sides of the tongue, often fissured 16) Which Rickettsial infection has a negative Weil-Felix test? 4
  • 5. A. Rickettsialpox (R. akari)Correct Choice B. Rocky Mountain Spotted Fever (R. rickettsii) C. Epidemic typhus (R. prowazekii) D. Mediterranean fever (R. conorii) E. Endemic typhus (R. typhi) Rickettsia are short, gram-negative rods which are strict intracellular parasites. The bacteria is transmitted via tick which much be attached for more than 6 hours. The Weil-Felix test exploits cross-reactivity between the Proteus vulgaris antigen and RIckettsia 17) A pool worker develops these painful erythematous nodules with ulceration on his fingers and then his arms. The antibiotic of choice to treat this infection is: A. Ceftriaxone B. Trimethoprim-sulfamethoxasole C. MinocyclineCorrect Choice D. Dapsone E. Penicillin Sporotrichoid spread may be seen in cutaneous leishmaniasis, actinomycosis, atypical mycobacterial infections, deep fungal infection, and melanoma. Mycobacterium marinum is associated with skin injury followed by exposure to contaminated water, usually from an aquarium, lake, or pool. The treatment of choice for M. marinum infections is minocycline 18) A 20-month old child develops a high fever followed 2 days later by a sudden eruption of rose pink macules and papules with white halos as the fever subsides. What is the most likely diagnosis? A. Measles B. Rubella C. Exanthem Subitum Correct Choice D. Erythema infectiosum E. Scarlet fever The cutaneous features of Exanthem Subitum (Roseola infantum, Sixth Disease)as described in the question develop as the fever subsides. While the other diagnoses are part of the differential diagnosis of "fever and rash" in children, only roseola infantum has this characteristic clinical course 19) WHO recommendations for the treatment of multibacillary leprosy include all of the following EXCEPT: A. Clofazimine B. All of these answers are recommended for the treatment of multibacillary leprosy. C. Rifampin D. Dapsone E. ClarithromycinCorrect Choice Clarithromycin is not part of the WHO recommendations for the treatment of multibacillary leprosy 5
  • 6. 20) The most likely cause of the childhood infection shown is: A. E. coli B. Pseudomonas Aeruginosa C. Group A Streptococcus Correct Choice D. S. Aureus E. H. influenza This is an example of perianal streptococcal “cellulitis.” The causative organism is Group A streptococcus 21) In addition to Kaposi’s sarcoma, HHV-8 infection is also the causative agent in: A. Pyogenic granuloma B. Pityriasis rosea C. Angiosarcoma D. Primary effusion lymphomaCorrect Choice E. Bacillary angiomatosis Human herpes virus 8 is a double stranded DNA virus. It is thought to be pathogenic in Kaposi’s sarcoma, primary effusion lymphoma, and Castleman’s disease. Primary effusion lymphoma is a rare B-cell lymphoma seen predominantly in patients with AIDS 22) The nonvenereal treponematosis shown is caused by which of the following organisms? A. T. pertenue B. T. pallidum (subsp. endemicum) C. B. duttonii D. T. carateum Correct Choice E. T. pallidum The depigmented patches on the hands shown are characteristic of the late (tertiary) stage of pinta. The causative organism is T. carateum 23) A 52 year-old butcher presents with a tender lesion on his left hand as shown. The treatment of choice for this condition is: A. Chloramphenicol B. Ciprofloxacin C. Tetracycline D. Penicillin Correct Choice E. Azithromycin The clinical description and lesion shown in the image suggest a diagnosis of erysipeloid, caused by Erysipelothrix rhusiopathiae. The treatment of choice is penicillin 24) The treatment of choice of cat-scratch disease is: 6
  • 7. A. Itraconazole B. Mebendazole C. Ceftriaxone D. Trimethoprim-sulfamethoxazole E. ErythromycinCorrect Choice Cat-scratch disease is caused by Bartonella henselae, which is spread among cats by fleas. The hallmark of the disease is unilateral and regional lymphadenopathy. The treatment of choice for cat- scratch disease is erythromycin 25) Pretibial fever is caused by: A. Listeriosis B. Rickettsiae spp. C. LeptospirosisCorrect Choice D. Rickettsiae spp. E. Ehlichiosis Leptospira autumnalis causes pretibial fever (“Fort Bragg fever” or anicteric leptospirosis), an acute exanthematous infectious erythema, generally most prominent on the shins. Leptospira interrogans, serotype icterohaemorrhagiae, is the most common cause of Weil’s disease or icteric leptospirosis 26) Guarneri bodies are associated with: A. Smallpox Correct Choice B. Anthrax C. Measles D. Orf E. CMV Guarnieri’s bodies are cytoplasmic eosinophillic inclusions found on light microscopy 27) Petechial lesions at the junction of the hard and soft palate are characteristically seen in which disease? A. Kawasaki’s disease B. Infectious mononucleosisCorrect Choice C. Scarlet fever D. Rocky Mountain spotted Fever E. Rickettsialpox infection Forsheimer spots are distinctive pinhead size petechiae found at the junction of the hard and soft palate. They may be seen in infectious mononucleosis 28) Congenital Varicella Syndrome occurs after maternal varicella infection during which stage of pregnancy? A. Third trimester 7
  • 8. B. First 20 weeks Correct Choice C. 20-24 weeks D. 5 days before and 2 days after delivery E. None of these answers are correct Congenital Varicella Syndrome occurs after maternal varicella-zoster virus infection early in pregnancy (up to 20 weeks gestation) 29) Of the choices listed, which antiobiotic is the best to treat a 7-year old child with erythema migrans? A. Tetracycline B. Azithromycin C. AmoxicillinCorrect Choice D. Doxycycline E. Rifampin Doxycycline is the usual treatment for erythema migrans in adults in the absence of neurologic or cardiovascular complications. In children less than 8, all tetracyclines are relatively contraindicated because of they can cause tooth discoloration. Therefore amoxicillin should be used as the first line therapy for children less than 8. Note: Changes have been made to the treatment of RMSF. Children of any age should be treated with doxycycline for that disease 30) A 48 year-old woman presents to the Emergency Room with a 2 day history of fever and a well demarcated erythematous plaque on her right cheek. What is the treatment of choice? A. Potent topical corticosteroid B. Fluconazole C. Intravenous penicillinCorrect Choice D. Prednisone E. Cephelexin The most likely diagnosis is Erysipelas which is caused by beta-hemolytic group A streptococcus. The treatment of choice is intravenous penicillin 31) Herpes simplex virus infection and this condition has been linked to which HLA type? A. HLA B15Correct Choice B. HLA B51 C. HLA DR3 D. HLA Cw6 E. HLA B27 HSV related erythema multiforme has been associated with an increased frequency of HLA-B15 32) Milker’s nodule is caused by infection with which of the following viruses? A. Orf virus 8
  • 9. B. Human herpesvirus-8 (HHV-8) C. Paravaccinia virusCorrect Choice D. Human herpesvirus-6 (HHV-6) E. Vaccinia virus Milker’s nodule is caused by paravaccinia virus, a poxvirus of the genus Parapoxvirus. It is transmitted to humans from infected cows 33) Which porphyrin is responsible for the fluorescence under Wood's lamp examination of erythrasma? A. Protoporphyrin IX B. Porphobilinogen C. Coproporphyrin IIICorrect Choice D. Coproporphyrinogen E. Uroporphobilinogen Erythrasma is caused by corynebacterium minutissimum. The presence of coprophyrinogen III created by the bacteria is responsible for the characteristic coral-red fluorescence under Wood's lamp 34) Epithelioma cuniculatum is an HPV-associated verrucous carcinoma involving the: A. Trunk B. Scalp C. Penis D. Oral mucosa E. SoleCorrect Choice Epithelioma cuniculatum is a form of verrucous carcinoma involving the sole 35) A 28 year-old man presents with fever, chills, arthralgias and the cutaneous lesions shown. The most likely diagnosis is: A. Gonoccemia Correct Choice B. Rickettsialpox C. Primary herpes simplex infection D. Anthrax E. Purpura fulminans The image shows sparsely distributed hemorrhagic vesiculopustules with erythematous bases on an acral surface. These features are characteristic of gonococcemia 36) Blueberry Muffin Baby can be a feature of all of the following EXCEPT: A. Rhabdomyosarcoma B. Langerhans’ cell histiocytosis 9
  • 10. C. TORCH D. Hemolytic disease of the newborn E. Klippel-Trenaunay-Weber syndrome Correct Choice Blueberry muffin lesions can be seen in the setting of prenatal infections (e.g. TORCH), severe anemia (e.g. Hemolytic Disease of the newborn) and neoplastic diseases (e.g. rhabdomyosarcoma). It is not associated with the Klippel-Trenaunay-Weber syndrome 37) The treatment of choice for Oroya Fever is: A. ChloramphenicolCorrect Choice B. Erythromycin C. Penicillin D. Minocycline E. Doxycycline The treatment of choice for Oroya Fever is chloramphenicol because of frequent superinfection with Salmonella 38) A 10 year-old with a 3-day history of fever and headache presents to the clinic with multiple erythematous papulopustules on his extremities, many of which have central eschars. The most likely diagnosis is: A. Erlichiosis B. RickettsialpoxCorrect Choice C. Meningococcemia D. Lyme disease E. Rocky mountain spotted fever Rickettsialpox is an acute febrile illness caused by the bacteria Rickettsia akari, a member of the spotted fever group. R. akari is transmitted by the house mouse mite, Liponyssus sangineus. Clinically, patients have an eschar at the inoculation site with a subsequent papulovesicular rash, fevers, headache, chills, diaphoresis, myalgia, and anorexia 39) A 35 year-old woman presents with a painful vaginal erosion and tender left-sided inguinal lymphadenopathy. Gram stain reveals clusters of coccobacilli in a “school of fish” pattern. What is the most likely diagnosis? A. Granuloma inguinale B. ChancroidCorrect Choice C. Primary syphilis D. Secondary syphilis E. Lymphogranuloma venereum Chancroid classically presents with a soft, tender chancre with ragged edges and unilateral, tender inguinal adenopathy (bubo). A “school of fish” pattern on Gram or Giemsa stain is diagnostic 40) All of the following are features of scarlet fever except: 10
  • 11. A. S. aureus infectionCorrect Choice B. Pastia’s lines C. Pharyngitis D. Circumoral pallor E. Sandpaper-like texture Scarlet fever is primarily a disease of children with most cases occurring between the ages of 1 and 10 years of age. Streptococcus pyogenes is the causative organism. It produces the streptococcus pyrogenic exotoxin which elicits the cutaneous manifestations by enhancing delayed type hypersensitivity to streptococcal products. Clinical findings include fever, sore throat, headache, chills, sandpaper rash on the trunks, strawberry tongue, and Pastia’s lines (linear petechial streaks found in flexural locations 41) Hebra nose deformity is characteristic of infection with which organism? A. Serratia marcesens B. Haemophilus influenzae C. Streptococcus pyogenes D. E. coli E. Klebsiella pneumoniaeCorrect Choice Hebra nose deformity is seen in Rhinoscleroma which is caused by Klebsiella pneumoniae 42) Pediculus humanus corporis can transmit: A. Scrub typhus B. Oriental typhus C. Epidemic typhusCorrect Choice D. Endemic typhus E. Murine typhus The body louse or Pediculus humanus corporis transmits Rickettsia prowasekii the organism responsible for epidemic typhus. Endemic typhus or murine typhus is caused by R. typhi that is transmited by the rat flea/Xenopsylla cheopis. Scrub typhus is caused by R. tsutsugamushi and is transmitted by chiggers or trombiculid mite larvae 43) Xenopsylla cheopis transmits: A. Rickettsia akari B. Epidemic typhus C. Scrub typhus D. Murine typhusCorrect Choice E. Rickettsia prowazekii Xenopsylla cheopis along with Ctenocephalides felis are fleas that transmit R. typhi, the organism responsible for murine or endemic typhus. The body louse or Pediculus humanus corporis transmits Rickettsia prowasekii the organism responsible for epidemic typhus. Scrub typhus is caused by R. tsutsugamushi and is transmitted by chiggers or trombiculid mite larvae 11
  • 12. 44) A 4 year-old boy is diagnosed with Staphylococcal Scalded-Skin Syndrome. Vesiculation in this disease is secondary to exfoliative toxins binding what target protein? A. Desmoglein 1Correct Choice B. Bullous Pemphigoid antigen 1 C. Bullous pemphigoid antigen 2 D. Desmoglein 3 E. Plectin Staphylococcus Scalded-Skin Syndrome (SSSS) is caused by Staphylococcus exfoliative toxins A and B binding desmoglein A. This results in the intraepidermal split at the granular layer seen in this illness. Targeting of desmoglein 3 would result in a suprabasilar split. Targetting of plectin, BPAG1, or BPAG2 would result in a subepidermal split. This is not seen in SSSS 45) Streptococcus Iniae has been shown to cause: A. Perianal dermatitis in neonates B. Bullous impetigo C. Perineal erysipelas in postpartum women D. Necrotizing fasciitis E. Hand cellulitis in fish handlersCorrect Choice Streptococcus iniae has been demonstrated to cause hand cellulitis from puncture wounds sustained form the dorsal fin, fish bone or knife of usually a tilapia. Treatment with PCN is curative. Group A Streptococci are the most common cause of perianal dermatitis. Many different bacteria have been implicated in necrotizing fasciitis. Bullous impetigo is most frequently caused by phage type 71 S. aureus or a related group 2 phage type. Group B streptococcus is most often responsible for perineal erysipelas in postpartum women. 46) What is the most common site of infection from Streptococcus Iniae? A. Face B. HandCorrect Choice C. Lower leg D. Foot E. Nails Streptococcus iniae most commonly causes bacteremic cellulitis of the hand in persons who have recently handled fresh fish. S. iniae is a fish pathogen that causes sporadic infection in tilapia, yellowtail, rainbow trout, and coho salmon. Treatment of choice is with penicillin antibiotics 47) The vector of New World Leishmaniasis is the: A. Deer fly (Chrysops spp.) B. Tsteste fly C. Phlebotomus sandfly D. Lutzomyia sandflyCorrect Choice E. Simulium black fly 12
  • 13. The sandfly belonging to the genus Lutzomyia is the vector of New World Leishmaniasis. Sandflies of the genus Phlebotomus are the vectors of Old World Leishmaniasis 48) How long is the life cycle for the mite that causes this infestation? A. 1 day B. 1 monthCorrect Choice C. 6 months D. 1 year E. 1 week Scabies is caused by the mite Sarcoptes scabiei var. hominis. The life cycle of the mite is 30 days. A female mite will lay 60-90 eggs during her life 49) Herpes-associated erythema mutliforme is most commonly associated with which HLA type? A. HLA DR2 B. HLA DR3 C. HLA B15Correct Choice D. HLA B8 E. HLA B27 Recurrent erythema multiforme (EM minor) is usually caused by recurrent herpes simplex, most commonly HSV-1 orolabial disease. This is more correctly now called herpes-associated erythema multiforme(HAEM) and has been more commonly associated with people with HLA type B15 50) All of the following are features of the Ramsay Hunt Syndrome EXCEPT: A. Infection of the geniculate ganglion B. Vesicles on the external ear C. Tinnitus D. Facial paresis E. Herpes simplex infectionCorrect Choice Ramsay Hunt syndrome results from varicella zoster virus infection of the geniculate ganglion of the seventh cranial nerve (CN VII). It is characterized by vesicles on the external ear or ear canal, tinnitus and/or other auditory symptoms, and ipsilateral facial paresis 51) Exanthem Subitum is caused by which of the following? A. Group A Steptococcus B. Coxsackie virus C. Human Herpes Virus-6 (HHV-6) Correct Choice D. Parvovirus B19 E. Epstein-Barr Virus Exanthem Subitum (Roseola Infantum, Sixth Disease) is caused by human herpesvirus 6, and 7 (HHV-6, HHV-7 13
  • 14. 52) Carrion's disease is characterized by fevers, headaches and arthralgias and is accompanied by severe hemolytic anemia. Superinfection with which organism is the most frequent cause of death? A. Shigella B. Bartonella henselae C. SalmonellaCorrect Choice D. Bartonella bacilliformis E. Bartonella Quintana Carrion's disease (Oroya fever) is characterized by fevers, headaches and arthralgias and is accompanied by severe hemolytic anemia. Superinfection with Salmonella is the most frequent cause of death. Protection from sandfly bites is all-important 53) Dracunculiasis is typically acquired from: A. Soil B. Salt water C. Black flies D. Drinking water Correct Choice E. Chiggers Dracunculiasis, which is caused by Dracunculus medinensis, is acquired by ingestion of copepod- infested water 54) Which subtype of the human papilloma virus is most likely to have caused this infection? A. HPV, type 3 B. HPV, type 6Correct Choice C. HPV, type 1 D. HPV, type 5 E. HPV, type 2 The human papilloma virus is a member of the Papovavirus family, a double-stranded DNA virus. HPV, type 6 has been implicated in giant conduloma of Buschke and Lowenstein and anogenital condyloma 55) Which of the following diseases is caused by a virus? A. Boutonneuse spotted fever B. Colorado tick feverCorrect Choice C. Q fever D. Rocky Mountain spotted fever E. Human monicytic ehrlichiosis Rocky Mountain spotted fever is caused by Rickettsia rickettsii. Human monicytic ehrlichiosis is caused by Ehrlichia chaffeensis. Colorado tick fever is caused by Reoviiridae. Boutonneuse fever is caused by Rickettsia conorii. Q fever is caused by Coxiella burnetti 14
  • 15. 56) Epidermodysplasia verruciformis is caused by which human papilloma virus (HPV) types? A. 3 and 10 B. 6 and 11 C. 5, 8, and 9Correct Choice D. 13 and 32 E. 16 and 18 Epidermodysplasia verruciformis is caused by HPV types 5, 8, 9, 12, 14, 15, 17, 19-26 57) Verruga peruana is transmitted by: A. Fecal oral contact B. Blackfly C. SandflyCorrect Choice D. Tick E. Fecal fecal contact Verruga peruana, a disease endemic to Peru and a few neighboring countries, is caused by Bartonella bacilliformis. It is transmitted by the sandfly, Lutzomyia verrucarum. It is preceded by an acute febrile stage called Oroyo fever or Carrion’s disease. The treatment of choice is chloramphenicol 58) Each of the following is true regarding leishmaniasis except: A. Arthropod vector is the sand fly B. Mazzotti’s test is diagnosticCorrect Choice C. Cutaneous disease is the most common form D. Pentavalent antimony used for visceral disease E. Espundia seen mucocutaneous disease Leishmaniasis is a parasitic infection born by a sandfly vector. No treatment is necessary for cutaneous leishmaniasis, pentavalent antimony for visceral leishmaniasis. Mazzotti reaction is used to test for onchocerciasis in which a single dose of dimethycarbamazine is given for reaction 59) What is the principal vector of Lyme Disease in the Northeastern U.S.? A. Lxodes ricinus B. Dermacentor variabilis C. Lxodes damminiCorrect Choice D. Ambylomma americanum E. Soft-bodied ticks (Ornithodoros) The proncipal vector of Lyme Disease in the Northeastern U.S. is Ixodes dammini (Ixodes scapularis 60) Which of the following mycobacterial organisms is classified as a photochromogen? 15
  • 16. A. M. tuberculosis B. M. grodonae C. M. KansasiiCorrect Choice D. M. ulcerans E. M. fortuitum M. Kansasii, M. marinum, and M. simiae are photochromogens. They form pigment when exposed to light 61) The vector of Trench Fever is the: A. Cat flea (Ctenocephalides felis) B. Human body louse (Pediculus humanus corporis)Correct Choice C. Trombiculid mite D. Sandfly (Phlebotamus perniciosus) E. Rat flea (Xenopsylla cheopis) The human body louse (Pediculus humanus corporis)is the vector of Trench Fever, Epidemic Typhus, and Relapsing Fever 62) Pediculus humanus var. corporis (human body louse) is the vector in which of the following diseases: A. Q fever (C. burnetii) B. Endemic typhus (R. typhus) C. Rickettsialpox (R. akari) D. Epidemic typhus (R. prowazekii)Correct Choice E. Rocky Mountain Spotted Fever (R. rickettsii) The human body louse is the implicated Trench fever, epidemic typhus, and relapsing fever. Trench fever usually affects alcoholic men and is manifested by fevers. Treatment is with ceftriaxone, erythromycin, or doxycycline. Epidemic typhus is manifested by fevers, chills, malaise, and a pink macular eruption beginning in the axilla and trunk. Treatment is with tetracycline or chloramphenicol. Relapsing fever is manifested by paroxysmal fevers, headache, lymphocytoma, and erythematous/petechial macules on trunk/extremities. Treatment is with doxycycline 63) A 27 year-old man with a 1-month history of bilateral inguinal lymphadenopathy and a positive “groove sign.” What is the most likely diagnosis? A. Gonorrhea B. Lymphogranuloma venereumCorrect Choice C. Syphilis D. Chancroid E. Herpes simplex Inguinal adenopathy with fluctuant, tender nodes above and below Poupart’s ligament - referred to as “groove sign” - is a characteristic clinical feature of lymphogranuloma venereum 16
  • 17. 64) The most common location of herpes gladiatorum is the: A. FaceCorrect Choice B. Anterior thigh C. Groin D. Hand E. Chest The face is the most common location for Herpes Gladiatorum 65) Which of the following is the vector of lymphatic filariasis caused by Wuchereria bancrofti? A. Black fly B. Deerfly C. Mosquito Correct Choice D. Botfly E. Tsteste fly Lymphatic filariasis is spread by mosquitoes belonging to genera Aedes, Anopheles, Culex, or Mansonia 66) What is the classic CXR finding of a patient with inhalational anthrax? A. Pleural effusions B. Increased interstitial markings C. Widened mediastinumCorrect Choice D. Alveolar infiltrates E. Normal x-ray Inhalation anthrax is the most lethal form of anthrax. Other forms include cutaneous anthrax and GI anthrax. The classic radiographic finding is a widened mediastinum 67) What is the vector for this nematodal infection? A. Tsetse fly (Glossine morsitans) B. Mango fly (Chrysops)Correct Choice C. Black fly (Simulium) D. Water flea (Cyclops) E. Reduviid bug Loiasis is caused by the nematode loa loa. It is transmitted by the mango fly (Chrysops dimidia or CHrysops silacea). It may manifest with painful, localized subcutaneous nonpitting edema called calabar swellings. Conjunctival migrations are also common 68) A goatherder develops this painless pustule that subsequently forms a black eschar. What causes the edema to form in this lesion? 17
  • 18. A. Inhibition of phagocytosis B. C1 esterase inhibitor C. Increased levels of cAMPCorrect Choice D. Release of TNF-alpha E. Release of IL-1beta Anthrax is caused by Bacillus anthracis, a gram positive spore-forming rod. Anthrax is primarily seen in individuals who are in contact with wild or domestic animals. Recent cases have been linked with bioterrorism. B. anthracis produces edema toxin and lethal toxin. Edema toxin is comprised of edema factor and protective antigen. Edema factor is a calmodulin dependent adenyl cyclase. The increased cAMP induces the gelatinous edema of anthrax skin lesions 69) Which human papillo viurs (HPV) type is implicated in papillomatosis cutis carcinoides di Gottron? A. 7 B. 11Correct Choice C. 60 D. 18 E. 57 Papillomatosis cutis carcinoides di Gottron, also know as Gottron's tumor, is a verrucous carcinoma of the skin. In 1948, Ackerman first described verrucous carcinoma (VC), a low-grade tumor that generally is considered a clinicopathologic variant of squamous cell carcinoma. Verrucous carcinomas are caused by HPV types 6 and 11. When they occur on the feet they are called epithelioma caniculatum. When they occur on the genitals they are refered to as Buschke- Lowenstein tumors. When they occur in the mouth they are called Ackerman's tumors or oral florid papillomatosis 70) A 48 year-old man develops headache, myalgias, and high fever 5 days after a hunting trip. On examination he has a tender ulcer with raised margins and an eschar on his index finger, as well as tender axillary lymphadenopathy. The most likely diagnosis is: A. Orf B. Anthrax C. Tularemia Correct Choice D. Glanders E. Erysipeloid Tularemia, which is caused by Francisella tularensis, occurs after exposure to infected animals, including, rabbits, foxes, and squirrels. The clinical description is that of the ulceroglandular form, which is the most common presentation 71) Which exotoxin(s) are involved in mediating the effects of Bacillus anthracis - the causative organism for anthrax? A. Lethal Toxin B. Edema Toxin and Lethal ToxinCorrect Choice C. Exotoxin B 18
  • 19. D. Edema Toxin E. Alpha toxin B. anthracis has 2 exotoxins: edema toxin and lethal toxin, each comprised of a pair of noncovalently linked proteins: – Edema toxin = Edema Factor (EF) + Protective Antigen (PA) – Lethal toxin = Lethal Factor (LF) + PA – EF causes gelatinous edema of anthrax skin lesions by inducing an increase in cyclic adenosine monophosphate (cAMP) levels – LF causes shock and death in disseminated anthrax via the release of tumor necrosis factor-alpha (TNF-a)and interleukin-1b (IL-1b) – PA is responsible for entry of exotoxins into the cell by receptor-mediated endocytosis 72) A 43 year-old man with uncontrolled HIV disease presents with yellowish, red papules, draining sinuses and ulcers perianally and periurethrally. Which histopathological or immunohistochemical stain may aid in diagnosis? A. Von Gieson (EVG) B. CD3 C. Steiner D. CD20 E. Von KossaCorrect Choice The most likely diagnosis is malakoplakia. Malakoplakia, an infection usually attributed to S. aureus, P. aeruginosa, or E. coli is characterized histopathologically by Michelis-Gutmann bodies. These are foamy eosinophilic macrophages containing calcified, laminated, intracytoplasmic bodies. They can be highlighted by a calcium stain such as von Kossa. CD3 and CD20 are lymphocyte markers, von Gieson is an elastic tissue stain, and Steiner stain is used to highlight spirochetes. These would not aid in the diagnosis of malakoplakia 73) A 42 year-old woman presents with a large, vegetating ulcer involving her left labia majora and groin for over 1 year. A Giemsa’s stained touch preparation reveals bipolar, safety pin-shaped intracytoplasmic inclusions. What is the most likely diagnosis? A. Chancroid B. Lymphogranuloma venereum C. Gonorrhea D. Primary herpes simplex E. Granuloma InguinaleCorrect Choice Bipolar, safety pin-shaped intracytoplasmic inclusions on Giemsa stain - known as Donovan bodies - are diagnostic of granuloma inguinale 74) The leading infectious cause of deafness and mental retardation in the U.S. is: A. Congenital herpes simplex B. Congenital syphilis C. Congenital Cytomegalovirus (CMV)Correct Choice D. Congenital varicella 19
  • 20. E. Congenital Rubella Congenital CMV is the leading infectious cause of deafness and mental retardation in the U.S. 75) Which human papillomavirus type is associated with giant condylomata of Bushke and Lowenstein (Bushke-Lowenstein tumor)? A. 2 B. 31 C. 16 D. 1 E. 6 Correct Choice The Bushke-Lowenstein tumor is caused by HPV-6 and 11 76) Each of the following species may be involved in necrotizing fasciitis except: A. MycobacteriumCorrect Choice B. Pseudomonas C. Bacteroides D. Streptococcus E. Clostridium Necrotizing fasciitis is a rapidly advancing acute necrotizing infection that may follow trauma, surgery, or occur spontaneously. It is associated with systemic toxicity and high mortality rate. Clinically, erythema, edema, and edema progresses to dusky cyanosis, blistering and necrosis. Many virulent bacteria have been culturesd including Pseudomonas, Bacteroides, streptococci, staphylococcus, enterococci, and clostridium. Both aerobic and anaerobic cultures should be taken. Mycobacterium is not associated with necrotizing fasciitis 77) Bacillary angiomatosis is caused by which of the following organisms? A. B. quintana B. All of these answers are correct C. B. quintana and B. henselaeCorrect Choice D. B. bacilliformis E. B. henselae B. quintana or B. henselae are both causative organisms for bacillary angiomatosis 78) A 24 year-old man from New York City develops fever, chills, malaise, and a severe headache. His apartment has a mouse problem. On examination, multiple, discrete papulovesicles and a single eschar are found. What is the most likely diagnosis? A. RickettsialpoxCorrect Choice B. Rocky Mountain Spotted Fever C. Q Fever D. Meningococcemia 20
  • 21. E. Ehrlichiosis The clinical description is that of rickettsialpox, which is caused by R. akari and spread by liponyssoides sanguineus - the house mouse mite 79) Nonmenstrual cases of the staphylococcal toxic shock syndrome are mediated by which of the following toxins? A. Epidermolytic toxins A and B B. Edema toxin C. Pyrogenic exotoxins A, B, C D. Exfoliative toxins A and B E. Enterotoxins B and C Correct Choice Toxic shock syndrome (TSS) is mediated by Enterotoxins B and C (50% of nonmenstrual cases) and TSS toxin 1 (TSST-1), which is the leading cause of menstrual cases 80) A 27 year-old HIV positive presents with this pruritic eruption? What is the characteristic cell type found on histopathology? A. Plasma Cell B. Lymphocyte C. Neutrophil D. EosinophilCorrect Choice E. Neutrophil Eosinophilic pustular folliculitis is a pruritic disorder that may be seen in HIV positive patients. Typically, the CD4 count is < 300. In patients with HIV, pruritus tends to improve after institution of anti-retroviral therapy 81) Which of the following statements best describes the pathogenesis of scrofuloderma? A. Autoinoculation of M. tuberculosis from advanced pulmonary tuberculosis B. Hematogenous spread of M. tuberculosis from a distant site of infection C. Primary incoculation of M. tuberculosis in a sensitized host D. An cutaneous manifestation of military tuberculosis E. Contiguous spread from an underlying focus of tuberculous lymphadenitis Correct Choice Scrofuloderma is the result of contiguous spread onto skin from an underlying focus of tuberculous infection. This typically occurs in a sensitized host with low immunity 82) Focal epithelial hyperplasia (Heck’s Disease) is associated with which of the following human papillomavirus types? A. 7 B. 13 Correct Choice C. 10 D. 4 21
  • 22. E. 3 Focal epithelial hyperplasia is caused by HPV-13 and 32 83) Winterbottom’s sign is a characteristic feature of which of the following diseases? A. Onchocerciasis B. Leishmaniasis C. Loaiasis D. Strongyloidiasis E. African Trypanosomiasis Correct Choice Winterbottom’s sign (posterior cervical lymphadenopathy) is a clinical feature of African trypanosomiasis. Romaña’s sign (eyelid edema and conjunctivitis at site of incoculation) is a diagnostic finding in American trypanosomiasis (Chagas’ disease) 84) A young patient presents to you after participating in a bar brawl a few nights before with a painful swollen erythematous right hand. The most likely organism is: A. Staphylococus aureus B. Bacteriodes fragilis C. Eikenella corrodensCorrect Choice D. Pasturella multocida E. Streptococcus pyogenes Eikenella corrodens is a member of normal oral flora, that is commonly the cause of human bite and fist fight infections. Pasturella multocida is a common organism found in dog and cat bites. Staph and Strep could be the right answers for a cellulitis, but with this history are not the correct choices. Baceriodes fragilis is found in normal bowel flora 85) Hutchinson’s teeth are a feature of which stage of syphilis? A. Primary B. Secondary C. Tertiary D. Early congenital E. Late congenital Correct Choice Hutchinson’s teeth (widely-spaced, peg-shaped upper incisors) are a seen in Late Congenital Syphilis (> 2 years of age 86) The causative organism in Whitmore disease is which of the following? A. Vibrio vulnificus B. Pseudomonas aeruginosa C. Streptobacillus moniliformis D. Burkholderia pseudomalleiCorrect Choice E. Klebsiella pneumoniae 22
  • 23. Whitmore disease also known as Meloidosis is characterized by pulmonary disease, septicemia, and miliary abscesses. It is caused by Burkholderia pseudomallei 87) All of the following are true regarding tularemia except: A. Potential for use as a biologic weapon B. Ticks serve as a reservoir C. No longer one of the CDC's list of reportable diseasesCorrect Choice D. Presents as an acute febrile illness E. Caused by the gram negative Francisella tularensis Tularemia is a zoonotic disease caused by the gram-negative coccobacillus Francisella tularensis. It is transmitted by arthropods (particularly tick bites) or from contact with infected mammals (usually rabbits). It characteristically presents as an acute febrile illness with other variable clinical manifestations depending on the route of inoculation. Possible symptoms could include an ulcer at the site of inoculation, pharyngitis, ocular lesions, lymphadenopathy, and pneumonia. Diagnosis can be made by culture or a fourfold titer change. Because of its potential use as a biologic agent, tularemia is once again a reportable disease 88) All of the following are potential causes of a false positive RPR except: A. Pregnancy B. Malaria C. Malignant MelanomaCorrect Choice D. Systemic Lupus Erythematosus (SLE) E. Lepromatous Leprosy Non-treponemal tests for syphilis measure antibodies against phospholipids released from treponemes and damaged host cells. False positive non-treponemal tests can occur in the setting of pregnancy, autoimmune diseases, other spirochete infections, and infectious diseases such as leprosy and malaria. It does not occur in cases of melanoma 89) Coxsackievirus A16 is implicated in the pathogenesis of which of the following dieases? A. Fifth disease B. Exanthem subitum C. Papular purpuric “gloves and socks” syndrome D. Hand-Foot-and-Mouth disease Correct Choice E. Erythema infectiousum Hand-Foot-and-Mouth disease is caused by coxsackievirus A16. Group A coxsackievirus infection is also associated with herpangina 90) The most common bacterial cause of purpura fulminans is: A. Group A streptococcus Correct Choice B. E. coli C. P. aeruginosa 23
  • 24. D. S. aureus E. H. influenzae Group A streptococcus is the leading bacterial cause of purpura fulminans 91) Which of the following is the causative organism of Tularemia? A. Orientia tsutsugamushi B. Klebsiella pneumoniae C. Burkholderia mallei D. Francisella tularensisCorrect Choice E. Streptobacillus moniliformis Tularemia is caused by infection with Francisella tularensis - most commonly acquired after contact with infected rabbits (e.g. hunters 92) Roseola infantum is caused by which virus? A. Pox virus B. Coxsackie virus A16 C. Parvovirus B19 D. Human herpes virus 6Correct Choice E. Epstein-Barr virus Roseola infantum may be cause be either human herpes virus types 6 or 7. It is the sixth of the traditional exanthems of childhood. A diffuse, maculopapular eruption is typically preceded by a prodrome of fever and either respiratory or gastrointestinal symptoms. The infection usually occurs in the spring and the fall and nearly all children are seropositive for the virus by the age of 4 93) Foamy macrophages containing Klebsiella pneumoniae are called: A. Mikulicz's cellsCorrect Choice B. Virchow cells C. Michaelis-Gutman cells D. Russell cells E. Hansemann's cells Mikulicz’s cells are foamy macrophages found in Rhinoscleroma – a chronic, inflammatory, granulomatous disease of the upper respiratory tract. Virchow cells are found in Hansen’s disease. Hansemann cells and Michaelis-Gutman bodies are found in Malakoplakia 94) The diagnosis of leishmaniasis can be confirmed by which of the following techniques? A. Culture in Sabourad's agar B. Warthin-Starry stain C. Culture in Michel's medium D. Weil-Felix test 24
  • 25. E. Culture in Novy-MacNeal-Nicolle (NNN) mediumCorrect Choice Confirmation of infection with Leishmaniasis is performed by culture in Novy-MacNeal-Nicolle (NNN) medium 95) What is the most common cause of focal epithelial hyperplasia? A. Pachyonychia Congenita B. human immunodeficiency virus(HIV) C. Dyskeratosis Congenita D. human papilloma virus(HPV)Correct Choice E. Ebstein Bar virus(EBV) Focal epithelia hyperplasia, also known as Heck's disease, is caused by HPV types 13, 32 and 57. It is most commonly found on the lower lip but also on the buccal mucosa, gums and tongue. It is mainly a disease of native Americans and Greenlander Eskimos. EBV causes an oral hairy leukoplakia in patients with AIDS. Pachyonychia Congenita patients commonly develop a benign oral leukoplakia, while Dykeratosis Congenita patients develop a premalignant oral leukoplakia 96) Which form of cutaneous tuberculosis is associated with the highest level of immunity? A. Tuberculosis gumma B. Tuberculosis verrucosa cutis C. Lupus vulgarisCorrect Choice D. Miliary tuberculosis E. Tuberculosis chancre Lupus vulgaris is an chronic, progressive form which usually occurs around nose, neck and earlobes, notable for gelatinous "apple-jelly" hue. Tuberculosis chancre and tuberculosis verrucosa cutis cutis are primary inoculations of tuberculosis infection. Tuberculosis gumma and miliary tuberculosis are associated with lowered states of immunity 97) Which of the following is the causative organism of cat-scratch disease? A. B. quintana B. B. burgdorferi C. B. bacilliformis D. B. henselae Correct Choice E. B. duttonii B. henselae is the cause of cat-scratch disease. Humans are infected by a cat bite or scratch, while the cat flea is responsible for cat to cat transmission 98) Herpangina is caused by which of the following? A. Parvovirus B19 B. Group A coxsackievirusCorrect Choice C. Human Herpesvirus 6 (HHV-7) 25
  • 26. D. Epstein Barr Virus E. Human Herpesvirus 6 (HHV-6) Herpangina is caused by Group A coxsackievirus 99) A patient on the bone marrow transplant service has a fever, neutropenia, hemorrhagic bullae and erythematous nodules with dusky gray centers. The organism most likely to be responsible is: A. Streptococcus pyogenes B. Pseudomonas aeruginosaCorrect Choice C. Mycobacterium tuberculosis D. Staphylococcus aureus E. Candida albicans Ecthyma gangrenosum is an infection of critically ill or immunocompromised individuals by Pseudomonas aeruginosa. Clinically, patients develop opalescent, tense vesicles or pustules surrounded by a narrow pink to violaceous halo. The lesions rapidly become hemorrhagic, then violaceous and necrotic leaving ulcers. The most common location is on the buttocks and lower extremities. Treatment is with aminoglycosides 100) What is the treatment of choice in a penicillin allergic individual? A. Rifampiin B. Penicillin C. DoxcycylineCorrect Choice D. Ceftriaxone E. Clindamycin Secondary syphillis is caused by the spirochete, T. pallidum. Penicillin is the treatment of choice for secondary syphillis. In penicillin allergic individuals, doxycyline may be used 101) A patient presents with this anesthestic plaque. Which cytokines are upregulated in this disease? A. IL-2Correct Choice B. All of these cytokines are upregulated C. IL-5 D. IL-10 E. IL-4 Leprosy is caused by infections to Mycobacterium leprae. It is a acid fast bacilli. In tuberculoid leprosy, Th1 cytokines (IL-2, IFN-gamma) are upregulated. Clinically, patients have few, well- circumscribed anesthetic plaques 102) A 43 year-old man presents with suppurative nodules and sinus tracts over the mandible. He also is noted to have poor oral hygiene. What is the most likely causative organism? A. Staphylococcus aureus B. Nocardia brasilensis 26
  • 27. C. Streptococcus somaliensis D. Bartonella Henselae E. Actinomyces israeliiCorrect Choice Actinomyces israelii is an anaerobic gram-positive rod which causes chronic suppurative nodules and sinus tracts with an exudate containing sulfur granules. It most commonly effects the cervicofacial area, especially near the mandible. The abdomen and thoracic areas can also be affected. The source of infection is endogenous and patients with poor oral hygiene, penetrating foreign bodies, and recent dental procedures are at increased risk. Treatment of choice is penicillin 103) A 32 year-old woman presents with meningitis and palpable purpura. A diagnosis of meningococcemia is confirmed via culture of cerebrospinal fluid. Which complement components are most likely to be deficient in this patient? A. C3 B. C2 C. C4 D. C5Correct Choice E. C1 Patients with meningococcemia often demonstrate deficiencies of late complement components C5-9. 104) A 62 year-old man presents with tinnitus, facial paralysis, and vesicles of the external ear. What is the most likely diagnosis? A. Primary Herpes Simplex infection B. Coxsackievirus A16 infection C. Cytomegalovirus infection D. Primary Varicella Infection E. Herpes Zoster InfectionCorrect Choice This patient has Ramsay Hunt Syndrome, herpes zoster infection of the geniculate ganglion. Vesicles can be seen on the tympanic membrane and the external ear. Symptoms include ipsilateral facial paralysis, tinnitus, or other auditory symptoms. Treatment is with acyclovir and systemic corticosteroids 105) Calabar swellings are a characteristic feature of which disease? A. Eumycetoma B. LoiasisCorrect Choice C. Carrion's Disease D. Tertiary Syphilis E. Onchocerciasis Calabar swellings are characteristic of Loiasis. They are transient, non-tender soft tissue swellings found most commonly around joints 106) What is the characteristic body on histopathology for this disease? 27
  • 28. A. Henderson-Patterson B. Cowdry Type ACorrect Choice C. Dutcher D. Guarnieri E. Cowdry Type B Herpes simlex virus is characterized by Cowdry type A bodies on histopathology. They appear as sharply demarcated eosinophilic structures separated by a clear halo from a basophilic rim of the infected cell's marginated nuclear chromatin. Cowdry B bodies may be seen in polio infections 107) A 10 year-old girl presents to the dermatology clinic with a pruritic eruption on her trunk after swimming at a beach in Florida. Of note, the lesions are limited to skin that was covered by her bathing suit. The most likely diagnosis is: A. Swimmer’s itch B. Cutaneous larva migrans C. Cercarial dermatitis D. Seabather’s eruption Correct Choice E. Swimming pool granuloma Seabather’s eruption characteristically involves areas covered by swimwear, as a result of coelenterate larvae becoming trapped underneath bathing suits. The causative organisms are larval forms of Edwardsiella lineata (sea anemone) and Linuche unquiculata (thimble jellyfish). 108) The Jarisch-Herxeimer reaction is most closely associated with the release of which of the following cytokines? A. IFN-alpha B. IL-10 C. IL-12 D. IL-4 E. TNF-alpha Correct Choice TNF-alpha. The Jarisch-Herxheimer reaction is associated with increased plasma concentrations of TNF-alpha after antibiotic treatment of syphilis. The Jarisch-Herxheimer reaction has also been described following treatment of louse-borne relapsing fever, lyme disease and leptospirosis 109) The causative organism of epidemic typhus is: A. Salmonella typhi B. R. prowazekiiCorrect Choice C. Francisella tularensis D. R. typhi E. R. akari Epidemic typhus is caused by infection with R. prowazekii 28
  • 29. 110) The expanding annular erythematous patch shown developed in a 25 year-old woman after returning from a weekend at her summer home in Long Island, New York. The vector of this infection is: A. Lutzomyia verrucarum B. Ixodes dammini Correct Choice C. Ixodes ricinus D. Ctenocephalides felis E. Pediculus humanus corporis This is an example of erythema migrans, seen in Lyme disease. Ixodes dammini is the principal vector in the northeastern U.S 111) Streptobacillus moniliformis is the causative organism of which infectious disease? A. Glanders B. Rat-bite fever (Haverhill fever)Correct Choice C. Tularemia D. Cat scratch disease E. Scrub typhus Rat-bite fever (Haverhill fever) is caused by Streptobacillus moniliformis 112) Which antibiotic is best to treat this condition caused by Bartonella hensalae. A. Ceftriaxone B. ErythromycinCorrect Choice C. Clindamycin D. Fluconazole E. Trimethoprim/sulfamethoxasole Bartonella hensalae is the causative agent of bacillary angiomatosis. The differential diagnosis of bacillary angiomatosis may include pyogenic granulomas and Kaposi's sarcoma. The treatment of choice is erythromycin 113) The treatment of choice of the tropical infectious disease shown is: A. Ivermectin B. Mebendazole C. Permethrin D. Pentavalent antimony Correct Choice E. Diethylcarbamazine The ulcerated nodule shown here is characteristic of cutaneous leishmaniasis (CL). Leishmaniasis is treated with pentavalent antimony 114) Tick-borne Relapsing Fever is caused by: 29
  • 30. A. B. duttoniiCorrect Choice B. B. henselae C. B. burgdorferi D. B. recurrentis E. B. bacilliformis Relapsing Fever can be either Louse-borne or Tick-borne. Tick-borne Relapsing Fever is caused by B. duttonii, while Louse-borne Relapsing Fever is caused by B. recurrentis 115) Cat scratch disease is caused by infection with: A. Bartonella speciesCorrect Choice B. Gram-positive bacilli C. Spirochetes D. Rickettsiae E. Gram-positive cocci Cat scratch disease is caused by infection with B. henselae - a gram-negative bacillus of the genus Bartonella 116) Corynebacterium minuttissimum is responsible for which of the following infections? A. Erysipeloid B. Trichomycosis axillarisCorrect Choice C. Majocchi's granuloma D. Erysipelas E. Malakoplakia Corynebacterium minutissimum is responsible for causing trichomycosis axillaris, a superficial bacterial overgrowth of axillary hairs characterized by circumferential yellow (most common), red, or black granular concretions. Majocchi's granuloma is a dermatophyte infection. Erysipelas is caused by beta-hemolytic group A streptococcus. Malakoplakia is usually caused by S. aureus, P. aeruginosa, or E. coli. Erysipeloid is caused by Erysipelothrix rhusiopathiae. Corynebacterium is also the causative organism in erythrasma and pitted keratolysis 117) Measles is caused by which type of virus? A. ParamyxovirusCorrect Choice B. Togavirus C. Parvovirus D. Picornavirus E. Rhabdovirus Measles is caused by an RNA virus known as paramyxovirus. The illness is characterized by high fever, cough, coryza, conjunctivitis, and Koplik's spots. Viral exanthem characterized by erythematous macules and papules beginning on the face and spreading caudally 3-4 days after prodromal symptoms 30
  • 31. 118) A 27 year-old woman who is 30 weeks pregnant presents with erythema migrans. The treatment of choice for this patient is: A. Doxycycline B. AmoxicillinCorrect Choice C. Erythromycin D. Clindamycin E. Chloramphenicol In the setting of pregnancy, amoxicillin is the treatment of choice for Lyme Disease since doxycycline is contraindicated 119) The treatment of choice for Loiasis is: A. Diethylcarbamazine (DEC)Correct Choice B. Permethrin C. Ivermectin D. Malathion E. Chloramphenicol The recommended treatment for loiasis is Diethylcarbamazine (DEC 120) The most likely target for exfoliative toxin A in bullous impetigo is: A. Desmoglein 1Correct Choice B. BPAG2 C. BPAG1 D. Desmocollin E. Desmoglein 3 Bullous impetigo is caused primarily by staphylococcus aureus phage type 71. This bacteria produces exfoliative toxin A binds to desmoglein 1 and produces produces the blisters characteristic of bullous impetigo 121) The infectious disease shown is most likely caused by which of the following organisms? A. L. mexicana complex B. L. infantum C. L. major D. Klebsiella pneumoniae rhinoscleromatis Correct Choice E. L. donovani The figure is an example of rhinoscleroma, which is caused by Klebsiella pneumoniae rhinoscleromatis. While the diagnosis of mucocutaneous leishmaniasis (MCL) could be considered, none of the leismania species listed as possible responses are causes of MCL 122) A 8 year-old girl presents to the pediatrician with a poorly marginated blue-red single indurated plaque on her right cheek. What is the most likely causative organism? 31
  • 32. A. Klebsiella pneumoniae B. Staphyloccus aureus C. Beta-hemolytic group A streptococcus D. Haemophilus influenzaeCorrect Choice E. Pseudomonas aeruginosa The most likely diagnosis is H. flu cellulitis. This infection usually occurs in children and is characterized by a single indurated plaque with poorly delineated margins. Erysipelas, an infection caused by beta-hemolytic streptococcus group A, usually presents with well delineated margins in adult patients 123) A 35 year-old woman who works in the animal product industry presents with a painless hemorrhagic bulla. A diagnosis of anthrax is suspected. Gram stain of the vesicle fluid should reveal which of the following? A. Gram-negative cocci B. Guarnieri bodies C. Gram-positive bacilli Correct Choice D. Gram-negative bacilli E. Gram-positive cocci Anthrax is caused by Bacillus anthracis, which is a gram-positive spore-forming rod 124) The mechanism of action of this virulence factor of Bacillus anthracis is via increasing the activity of tumor necrosis factor alpha: A. Lethal toxinCorrect Choice B. All of the above answers are correct C. Edema toxin D. Polyglutamate acid capsule E. None of the above answers are correct Bacillus anthracis is responsible for causing Anthrax. Anthrax of the skin is characterized by a painless edematous nodule which rapidly degenerates into an eschar. B. anthracis has 3 virulence factors: a polyglutamate acid capsule which inhibits phagocytosis of the bacteria, edema toxin (edema factor and protective factor) which results in edema secondary to induction of cAMP pathway, and lethal toxin (lethal factor and protective factor) which leads to shock and death via increasing TNFalpha activity 125) At what rate of speed does this parasite migrate through the skin? A. 0.1 cm/day B. 10 cm/day C. 1 cm/day D. 2 cm/dayCorrect Choice E. 100 cm/day 32
  • 33. The infection depicted is larva migrans or creeping eruption which is caused by Ancylostoma braziliense. This nematode is typically a hookworm of cats and dogs. It migrates at a rate of 2 cm/day. Larva currens migrates at a rate of 10 cm/day 126) High doses of which of the following antiviral agents has been associated with thrombotic thrombocytopenic purpura in immunosuppressed patients? A. Acyclovir B. ValcyclovirCorrect Choice C. Cidofovir D. Foscarnet E. Famciclovir Thrombotic thrombocytopenic purpura has been reported with high doses of valcyclovir in immunosuppressed patients 127) Orf virus infection is also known as: A. Ecthyma B. Pseudocowpox C. Ecthyma contagiosumCorrect Choice D. Milker's nodule E. Echtyma gangrenosum Orf virus infection is also known as Ecthyma contagiosum. Ecthyma Gangrenosum is caused by Pseudomonas aeruginosa. Ecthyma is caused by Staphylococcal or streptococcal pyoderma. Pseudocowpox and Milker's nodule are also known as paravaccinia and caused by udders of infected cows 128) Botryomycosis is: A. An acute, disseminated infection involving the genitourinary system B. A fungal infection caused by T. tonsurans C. A bacterial infection commonly associated with Staphylococcus aureusCorrect Choice D. A bacterial infection commonly associated with Clostridium botulinum E. A fungal infection caused by M. canis Botryomycosis is an uncommon, chronic, idonlent disorder characterized by nodular, crusted, purulent lesions. Sinuses that discharge sulfur granules are present. These heal with atrophic scars. The granules yield most commonly Staphylococcus aureus on culture, although cases caused by Pseudomonas aeruginosa, E. coli, Proteus, Bacteroides, and Streptococcus have been reported 129) Xenopsylla cheopis is the vector responsible for which infectious disease? A. Human Granulocytic Ehrlichiosis B. Scrub typhus C. Endemic typhusCorrect Choice D. Human Monocytic Ehrlichiosis 33
  • 34. E. Epidemic typhus Xenopsylla cheopis (rat flea) is the vector of endemic typhus, which is caused by R. typhi 130) Erythema infectiosum is caused by a: A. None of these answers are correct B. Single stranded DNA virusCorrect Choice C. Double stranded DNA virus D. RNA virus E. Herpes virus Erythema infectiosum is caused by parvovirus B19 which is a single stranded DNA virus 131) Which of the following vectors is responsible for transmitting Chagas’ disease? A. Reduviid bug Correct Choice B. Sandfly C. Black fly D. Tstse fly E. Mosquito The reduviid big is the vector of American trypanosomiasis (Chagas disease). The vector of African trypanosomiasis is the tstse fly 132) All of the following statements are true of Bacillus anthracis infection except: A. inhalation, GI and cutaneous forms exist B. ciprofloxacin and doxycycline are first line treatments C. Spores remain stable for decades D. 20% mortality rate in untreated cutaneous infectionsCorrect Choice E. no recommendations from CDC exist on anthrax vaccine Anthrax can present in three different clinical forms: cutaneous, inhalational, and gastrointestinal. The cutaneous form appears as a painless vesicle that later forms an eschar (malignant pustule). The cutaneous form has a mortality rate of less than 1% if treated an up to 20% if untreated 133) Which of the following is the vector responsible for the transmission of Verruga Peruana? A. Ctenocephalides felis B. Tsetse fly C. Lutzomyia verrucarumCorrect Choice D. Pediculus humanus corporis E. Simulium slossonae Verruga Peruana is caused by infection with B. bacilliformis, which is transmitted by the sandfly, Lutzomyia verrucarum 34
  • 35. 35